#7 Rosh Review

Pataasin ang iyong marka sa homework at exams ngayon gamit ang Quizwiz!

Question: What is a felon?

Answer: An infection of the pulp of the distal finger or thumb. Rapid Review Flexor Tenosynovitis Penetrating trauma → S. aureus infection Kanavel's cardinal signs ​Fusiform or symmetrical swelling of finger Flexed posture of finger Tenderness along flexor tendon Pain with passive range of motion ​ABX, surgical emergency

Question: What is the most common injury associated with traumatic hemarthrosis of the knee joint?

Answer: Anterior cruciate ligament (ACL) rupture. Rapid Review Meniscus Injuries Knee rotary stress Rapidly changing directions, squatting Medial > lateral Clicking, locking McMurray test Medial joint line pain: medial meniscus injury Lateral joint line pain: lateral meniscus injury Ege test: pain/click on knee rotation with patient in squatting position External rotation pain: medial meniscus injury Internal rotation pain: lateral meniscus injury Dx: MRI

Question: When obtaining cervical spine X-rays in the setting of trauma, which views should be obtained?

Answer: Anteroposterior, lateral, oblique, and odontoid. Rapid Review Jefferson/C1, Fracture: Burst fracture of C1 Unstable Axial load - diving into a pool, head to head tackle in contact sports Repulsed boney fragments cause the majority of spinal cord trauma

Question: List other medication classes used in the management of Meniere's disease?

Answer: Antiemetics, adrenergic agonists, benzodiazepines and antihistamines. Rapid Review Meniere's Disease ↑ Endolymph Recurrent vertigo + tinnitus + sensorineural hearing loss Abrupt onset Ear fullness Vestibular rehabilitation, low-salt diet, HCTZ, surgery

Question: Current guidelines recommend the MMR vaccination at what two ages?

Answer: Between 12-15 months and between 4-6 years. Rapid Review Pregnancy: Contraindicated Vaccines Mumps-Measles-Rubella Varicella (chicken pox) Yellow fever BCG Vaccinia

Question: Blockade of which receptor helps to reduce the incidence of extrapyramidal effects?

Answer: Blockade of muscarinic acetylcholine receptors suppresses the emergence of extrapyramidal effects. Rapid Review Extrapyramidal Symptoms Typical > atypical antipsychotics Acute dystonia: muscle spasms, stiffness, oculogyric crisis, rx: benztropine, dyphenhydramine Akathisia: restlessness, rx: benztropine, ßBs Bradykinesia: Parkinsonism, rx: benztropine Tardive dyskinesia: orofacial involuntary movements

Question: What is the deep-tendon response in patients with botulinum poisoning?

Answer: Botulinum toxin prevents the release of acetylcholine from motor neurons resulting in flaccid paralysis and diminished deep-tendon reflexes. Rapid Review Botulism Patient will be an infant With a history of eating honey Complaining of feeble cry, constipation PE will show symmetric descending paralysis ("floppy baby") Most commonly caused by Clostridium botulinum Treatment is IV botulism Ig

Question: What are some examples of newer oral anticoagulation agents?

Answer: Dabigatran, apixaban, riveroxaban, edoxaban.

Question: What is the lethal disease that results from the parvovirus in pregnancy?

Answer: Hydrops fetalis. Rapid Review Erythema Infectiosum (Fifth Disease) Patient will be a child With a history of URI symptoms 3 - 4 days prior to rash PE will show "slapped cheek" rash Most commonly caused by parvovirus B19 Comments: Aplastic crisis in sickle cell patients

Question: What are the recommendations for home monitoring of blood glucose?

Answer: It is best recommended for those on multiple insulin injections. In those only on oral medications, or those with well- controlled glycemia, home monitoring has questionable utility and benefit. Rapid Review Diabetes Mellitus Type 2 ↑ Insulin resistance Obesity, family hx Candidiasis Blurry vision Hyperosmolar hyperglycemic state Lifestyle changes First line pharmaceutical rx: metformin

Question: What are some causes of chronic laryngitis?

Answer: Laryngopharyngeal reflux, allergies, smoking, chronic vocal abuse and inhaled corticosteroids. Rapid Review Laryngitis Viral > bacterial Hoarseness Voice rest: no benefit

Question: What is the first clinical manifestation of magnesium toxicity?

Answer: Loss of deep tendon reflexes.

Question: What is the earliest sign of papilledema on funduscopic examination?

Answer: Loss of normal venous pulsations. Rapid Review Central Retinal Vein Occlusion Varying degree of vision loss Monocular vision loss Fundoscopic exam: "blood and thunder" Ophthalmology consultation

Question: What types of food or drink can bring on an attack of acute gout?

Answer: Meat, seafood, alcohol and foods high in purine.

Question: What is the organism responsible for invasive fungal infections on the face?

Answer: Mucormycosis caused most often by Rhizopus. Rapid Review Sinusitis Patient will be complaining of pain over sinuses PE will show purulent rhinorrhea Most commonly caused by viral URI Treatment is supportive care Comments: bacterial sinusitis - persistent symptoms for more than 10-14 days - amoxicillin-clavulanate

Question: What is the treatment of conduct disorder?

Answer: Multisystemic Treatment, an intensive integrative program that emphasizes correct behavior within the individual or the individual's family, instead of within society as a whole. Rapid Review Conduct Disorder Pediatric version of antisocial personality disorder Violating human rights of others

Question: What is the recommended treatment for moderate to severe obstructive sleep apnea?

Answer: Nasal continuous positive airway pressure. Rapid Review Obstructive Sleep Apnea Patient will be obese With a history of allergies Complaining of apneic episodes while sleeping, snoring and daytime sleepiness PE will show enlarged tonsils Diagnosis is made by sleep study Treatment is weight loss, CPAP Comments: can lead to pulmonary hypertension and cor pulmonale (right ventricular hypertrophy)

Question: Does a Still's murmur radiate to the carotids?

Answer: No.

Question: What lab abnormalities are used in the diagnosis of premenstrual syndrome?

Answer: None. Rapid Review Premenstrual Syndrome Onset: 1-2 weeks prior to cycle, resolves after menstruation begins Sleep disturbances ↓ Focus Emotional lability Breast tenderness, HA Does not hinder personal/professional life (unlike premenstrual dysphoric disorder) Initial rx: dec caffeine intake, exercise, stress reduction Pharmacologic rx: NSAIDs, SSRIs, OCPs

Question: Which conditions are commonly comorbid with acromegaly?

Answer: Obstructive sleep apnea, glucose intolerance and diabetes, congestive heart failure and colon polyps or carcinoma. Rapid Review Acromegaly Patient will be complaining of increased head, glove, or shoe size PE will show coarse facial features, oily skin, visual field deficits, diabetes Labs will show increased IGF-1 Most commonly caused by pituitary adenoma Treatment is transphenoid resection

Question: What is the initial diagnostic test used in evaluating dysmenorrhea?

Answer: Pelvic ultrasonography to investigate the presence of secondary causes. Rapid Review Primary Dysmenorrhea Occurs only during menstrual cycles ↑ PGF2α → ↑ uterine contractions NSAIDs, acetaminophen

Question: What is the difference between plegia and paresis?

Answer: Plegia refers to paralysis (flaccid, no movement) while paresis refers to weakness (movement, but abnormal strength). Rapid Review Transient Ischemic Attack Transient episode of neurological dysfunction without acute infarction 10% of TIA patients will have a stroke within 90 days Aspirin + dipyridamole ABCD2 score: predicts likelihood of subsequent stroke within 2 days

Question: Name three genetic disorders, and their respective protein abnormalities, that are associated with aortic dissection?

Answer: Polycystic kidney disease (polycystin), Marfan's syndrome (fibrillin) and Ehlers-Danlos IV (type 3 procollagen). Rapid Review Aortic Dissection Patient will be older With a history of HTN, smoking, Marfan syndrome Complaining of sudden "ripping" or "tearing" CP radiating to back PE will show asymmetric pulses/BP CXR will show widened mediastinum Diagnosis is made by CT or transesophageal echocardiogram (TEE) Treatment is reduce BP, surgery

Question: What is Dressler's syndrome?

Answer: Post-myocardial infarction pericarditis. Rapid Review Myocardial Infarction: ECG findings Earliest finding: hyperacute T waves ST elevation Reciprocal ST depression (PAILS: posterior → anterior → inferior → lateral → septal) T wave inversions New LBBB Sgarbossa Criteria for STEMI with LBBB Concordant ST elevation > 1 mm in leads with a positive QRS (5) Concordant ST depression > 1 mm in V1-V3 (3) Discordant ST elevation ≥5 mm in leads with a negative QRS (2) ≥3 = STEMI

Question: What is the characteristic finding consistent indicative of corneal ocular involvement in zoster ophthalmicus?

Answer: Pseudodendrites (no terminal bulb).

Question: How can purpura be differentiated from ecchymoses?

Answer: Purpura do not blanch under pressure. Rapid Review Henoch-Schonlein Purpura (HSP) Patient will be 4-12 years old With a history of recent URI Complaining of abdominal pain, arthralgia, and a rash that began on buttocks and lower extremities PE will show maculopapular rash (palpable purpura) that is non-pruritic Most commonly caused by IgA mediated vasculitis Treatment is supportive care Comments: Complications include nephropathy, intussusception

Question: What are the common findings found on fundoscopy in central retinal artery occlusion?

Answer: Retinal edema with a pale appearance and a cherry-red spot representing the fovea. Rapid Review Acute Angle-Closure Glaucoma Patient will be entering a dark room or movie theater Complaining of acute unilateral painful vision loss, vomiting, and seeing halos around lights PE will show cloudy cornea and fixed mid-dilated pupil Labs will show ↑ IOP ( > 21 mm Hg) Treatment is topical ßBs, carbonic anhydrase inhibitors, steroids, miotics

Question: What is the most common type of bursitis?

Answer: Subacromial (subdeltoid) bursitis. Rapid Review Olecranon Bursitis Swelling, pain ↓ Active ROM Full passive ROM NSAIDs

Question: What treatment can be curative in constrictive pericarditis?

Answer: Surgical pericardectomy.

Question: What is one location of osteomyelitis in adults where hematogenous seeding is the most common mechanism of infection?

Answer: Vertebral osteomyelitis. Genitourinary and respiratory tract infections, IV drug abuse, and sickle cell disease all increase the risk of vertebral osteomyelitis. Infection is usually via hematogenous seeding of bacteria. Rapid Review Osteomyelitis Adults: contiguous spread Children: hematogenous spread Most common: S. aureus Sickle cell: Salmonella Cat/dog bites: Pasteurella multocida Plain films: periosteal elevation or bony erosions Bone scan, MRI Long term ABX

One Step Further Question: Given that African-Americans have a higher rate of mortality due to colorectal cancer, do the USPSTF guidelines apply to all races?

Answer: Yes.

Which of the following is prominently visualized in the image above? Haustra Plicae circulares Taneniae coli Valvulae conniventes

Correct Answer ( A ) Explanation: It is often difficult to decipher large bowel from small bowel on an X-ray. The air-filled large bowel commonly features a haustral pattern, as seen in the diagram below. The haustral folds are thicker than the valvulae conniventes of the small bowel. They also commonly do not appear to completely traverse the bowel. These haustral folds are caused by the contractions of the muscular taenia coli, longitudinal bands that span the large bowel. Another way to identify large bowel is that it usually contains feces. Generally, the small bowel lies centrally within the abdomen. Valvulae conniventes (D), also called plicae circulares (B), are thin, circular folds of mucosa, some of which are circumferential and are seen on an X-ray to pass across the full diameter of the lumen of the small bowel. Valvulae conniventes are also spaced closer together than the haustrations of the large bowel are. Taenia coli (C) are longitudinal bands of smooth muscle on the outside of the ascending, transverse, descending, and sigmoid colon whose contractions cause the appearance of haustra. The bands converge at the root of the vermiform appendix and the rectum.

A 58-year-old man with a history of obesity presents to your office with a complaint of daytime sleepiness. He says that his wife sleeps in a different room because of his snoring nighttime restlessness and getting up to urinate 2-3 times per night. Which of the following is the most appropriate diagnostic test? Polysomnography sleep study Post-void residual testing Pulmonary function tests Upper airway MRI

Correct Answer ( A ) Explanation: Obstructive sleep apnea (OSA) is the most common type of sleep-disordered breathing and is caused by collapse of the upper airway. Patients often present because of daytime sleepiness or because of complaints from their bed partner of snoring, gasping or interruptions in breathing during sleeping hours. Risk factors for OSA include advanced age, obesity and male gender. Physical exam may reveal a narrowed airway, large neck and waist circumference, and hypertension. OSA is not a clinical diagnosis and diagnostic testing is indicated for patients with excessive daytime sleepiness that cannot be explained. A sleep study with polysomnography in a laboratory setting is the recommended test to confirm the diagnosis of OSA. Post-void residual testing (B) is used for suspected urinary retention or bladder outlet obstruction, which are conditions to be considered for patients with nocturia. Pulmonary function tests (C) are not indicated when making a diagnosis of OSA. Upper airway MRI (D) and other diagnostic imaging tests are used as a research tool. Routine imaging is not used in the diagnosis of OSA.

A 44-year-old man presents with insidious and atraumatic proximal lower leg pain. It is more tender during rest than exercise. It commonly occurs at night, and doesn't seem to improve with oral analgesics. Knee testing is negative for meniscal or ligamentous injury. A screening radiograph reveals no fracture, but does show a non-speckled calcified lesion within the proximal tibia. A bone scan shows no spine or pelvic lytic lesions. Which of the following is the most likely diagnosis? Chondrosarcoma Enchondroma Multiple myeloma Osteoid osteoma

Correct Answer ( A ) Explanation: Primary malignant bone tumors are rare and benign bone tumors are common. However, metastatic bone tumors are very common, and skeletal metastasis must be strongly considered in any patient over 40 years of age with any bony lesion. The most common bone tumors in those over 40 years of age are metastases, chondrosarcoma, fibrosarcoma and multiple myeloma. Constant deep achy pain, especially in the nighttime hours and not relieved by rest, is the main symptom of bone malignancy. Large benign tumors may present in a similar fashion. Mass is a less common presenting complaint, but may suggest a benign condition especially if it is nontender. Chondrosarcoma and fibrosarcoma are common bone malignancies in adult metaphyseal bones. Osteosarcoma also has a metaphyseal predilection, however, it occurs with lytic and blastic lesions in the second decade, with chondrosarcoma having calcified lesions and fibrosarcoma having lytic lesions. Radiographic examination is necessary. CT is best used in evaluating benign bone lesions. MRI is better for evaluating malignant bone lesions. Bone scans are used to detect other sites of skeletal lesions. A chest radiograph and CT are usually ordered if a malignant bone tumor is suspected. Routine laboratory testing is of limited use, however, in those over 40 years of age, consider urinalysis, urine and serum protein electrophoresis and PSA testing. Benign tumors may be observed or excised, while malignant tumors are excised and treated with chemotherapy and radiotherapy. Enchondroma (B) is a benign cartilaginous tumor. It appears as speckled calcifications within the phalanges and metacarpals of young adults. Multiple myeloma (C) most commonly occurs in those over 40 years of age. Lytic lesions of the spine and pelvis are key findings. Osteoid osteoma (D) is a childhood and teenage benign bone lesion, commonly associated with night pain that is relieved with NSAIDs.

A 27-year-old man presents with painful, swelling of his fingertip for three days as shown above. He also reports subjective fever and malaise. Which of the following represents the best course of management? Analgesia and a clean dressing Bedside incision and drainage Intravenous antibiotic administration and hospital admission Operative open irrigation and debridement

Correct Answer ( A ) Explanation: The patient has herpetic whitlow. This is a localized infection caused by the herpes simplex virus. Inoculation most commonly occurs when the patient touches herpetic lesions on the mouth or genitals. Physical examination reveals grouped vesicles on an erythematous base. Analgesia and a clean dressing minimizes the chance of transmission to other people or other areas of the body. Topical acyclovir may decrease the length of illness and oral acyclovir may decrease the risk of recurrence, but this is controversial. Bedside incision and drainage (B) and operative open irrigation and debridement (D) should not be performed as this increases the risk of spread to other areas of the body or to other people. This infection is not caused by bacteria, and therefore intravenous antibiotic administration and hospital admission (C) is not effective.

You are treating a patient with type 2 diabetes mellitus. He is currently on metformin, but his hemoglobin-A1c is not at goal. You decide to add insulin. Which of the following is the best approach when considering this management plan? Continue oral medications, add long-acting insulin Continue oral medications, add short-acting insulin Stop oral medications, start long-acting insulin Stop oral medications, start short-acting insulin

Correct Answer ( A ) Explanation: There are several classes of medications for DM2 (see table below). Any DM2 treatment plan begins with education and lifestyle modification. Initial pharmacotherapy treatment of DM2 usually begins with an oral agent or combination oral therapy. If response is poor, many consider adding insulin. When initiating insulin therapy, oral medications should be continued initially. Long-acting insulin preparations are preferred as initial therapy. Long-acting insulin is preferred over short-acting insulin (B) in the early stages of insulin therapy. A gradual transition, and not abrupt cessation (C and D), of oral medications to insulin is the recommended approach.

A 58-year-old man with chronic hypertension presents to the ED with acute, 10/10 tearing substernal pain that radiates to the back. All you can gather from him is that he also has some type of "collagen disorder" and diabetes. A chest radiograph reveals a widened mediastinum. As you prepare for a transesophageal echocardiogram, you would most likely start which of the following medications as a first-line agent? Clonidine Labetalol Lisinopril Nitroprusside

Correct Answer ( B ) Explanation: A classic aortic dissection involves an intimal tear and hemorrhagic extravasation into the intima-media space. Aortic dissections can be defined as proximal, affecting the ascending aorta, or distal, involving the descending aorta distal to the subclavian take-off. Several risk factors of this potentially fatal condition exist, and include hypertension, congenital aortic valve disorder (coarctation, root-dilatation or bicuspid valve), trauma, cardiac surgery, aortitis and connective tissue disorder. Both types are characterized by severe, "ripping or tearing" pain that is maximal at onset (as compared to the crescendo pain of an acute coronary syndrome) and located in the chest, back or abdomen. A new aortic insufficiency murmur is more common in proximal dissection, while a history of hypertension is more common in the distal type. Evaluation includes chest radiograph, chest CT and transesophageal echocardiogram. First-line therapy is intravenous beta-blockers, such as labetalol or esmolol, followed by vasodilators like nitroprusside. Emergency surgery is very likely, especially for proximal dissections. Alpha-2-agonists, like clonidine (A), and angiotensin-converting enzyme inhibitors, like lisinopril (C), are not the recommended treatment of aortic dissection. The goal in treating aortic dissection is to first use beta-blockers, and then use vasodilators, like nitroprusside (D). Administering beta-blockers first helps to protect against reflex tachycardia and positive inotropy that sometimes occurs with agents that cause profound vasodilation.

A 77-year-old woman presents with 2 days of bowel incontinence, leg pain, and difficulty ambulating. Lower extremity examination reveals bilateral sensory deficits of the lumbosacral dermatomes, bilateral hyporeflexia, negative Babinski testing, and weak rectal tone. There is no lower extremity spasticity. Cranial nerve and upper extremity examination is normal. These symptoms are most likely the result of injury to which of the following structures? Lumbar disk Lumbosacral nerve roots Spinal cord gray matter Spinal cord white matter

Correct Answer ( B ) Explanation: A normal adult spinal cord ends as the conus medullaris at the L1-L2 level. The remaining L2-S4 spinal nerve roots that occupy the subdural space from the conus to the filum terminale are called the cauda equina. The cauda equina is a collection of post-ganglionic nerves (lumbosacral nerve roots), so injury to it presents as a lower motor neuron injury pattern with flaccid tone, paresis or paralysis, and hyporeflexia (or areflexia). Upper motor neuron symptoms such as hyperreflexia and clonus, spasticity and upgoing toes (positive Babinski test) are absent. The clinical presentation of cauda equina syndrome includes back pain, perianal numbness, loss of rectal tone, bowel and bladder (urinary retention followed by incontinence) abnormalities, leg weakness, loss of bulbocavernosus reflex, and loss of deep tendon reflexes. Cauda equina syndrome most commonly results from a herniated disk, but can also occur from trauma, an infection (epidural abscess), or a hematoma (epidural hematoma). A lumbar disk abnormality (A), such as herniation, typically causes unilateral radicular pain and symptoms, but it is almost exclusively associated with normal rectal tone and perineal sensation. Spinal cord tumor, or other spinal cord neuronal cell body disease affecting the gray matter (C), would cause upper motor neuron symptoms. Hyperreflexia, positive Babinski testing and spasticity are not present in this patient. Spinal cord neuronal axon or tract disease affecting the white matter (D), as in multiple sclerosis, usually presents earlier in age, and with upper extremity and cranial nerve neurologic findings, both of which are absent in this case.

A 28-year-old woman presents with difficult menses. She reports significant midline pelvic pain during the first two days of her regular menstrual cycles. The pain is so bad that she frequently misses work. Fortunately, her pain is self-limited, as the rest of her cycle is relatively comfortable. Which of the following is the most likely diagnosis? Amenorrhea Dysmenorrhea Mastodynia Metrorrhagia

Correct Answer ( B ) Explanation: A very common gynecologic complaint is painful menstruation, called dysmenorrhea. Most women have pain during menses, but dysmenorrhea is defined as pain that alters normal activity or requires pain medications to control. It is postulated that increased prostaglandins and leukotrienes are causative. There are three classes of dysmenorrhea: primary, which has no organic cause; secondary, which is associated with conditions such as endometriosis, pelvic inflammatory disease and intrauterine devices; and membranous, which is due to the rare passage of an endometrial cast through a constricted cervix. The typical pain questions are an important part of the history. Physical examination is usually unrevealing in primary, but not secondary, dysmenorrhea. This patient is menstruating. As such, amenorrhea (A), or the absence of menses, is incorrect. Mastodynia (C) is cyclical, painful breast swelling that occurs after ovulation, not during menstruation. Metrorrhagia (D) is light uterine bleeding that occurs at irregular intervals, and typically is not associated with significant pelvic pain.

A 52-year-old man presents with one month of a hoarse voice. He denies vocal trauma, recent upper respiratory infection, allergies, cough, rhinorrhea, malaise or fever. His past medical history includes tobacco abuse, hypertension, hyperthyroidism and osteoarthritis. Which of the following is the most appropriate management decision at this time? Home air humidification Laryngoscopy Speech-language therapy referral Tympanometry

Correct Answer ( B ) Explanation: Breathy, weak, raspy or rough voice quality is referred to as hoarseness, one cause of which is laryngitis. Acute laryngitis is a common condition presenting in the primary care clinic. Two of the most common causes are vocal abuse (e.g. excessive singing) and viral upper respiratory infection, which may be brought on by allergic irritation, prior sinusitis or prior rhinitis. The common reported history of viral laryngitis typically contains mention of allergies, cough, rhinorrhea, sneezing, excessive throat clearing and watery eyes. Empiric treatment, without further diagnostic evaluation, is recommended when symptoms last less than 2 weeks, there is an apparent benign cause like recent URI, and there is poor suspicion for malignant etiologies, such as carcinoma or prolonged tobacco use. If the underlying cause is not apparent, or if symptoms last longer than 2 weeks, or if symptoms do not resolve quickly with common treatments, laryngoscopy is recommended. Empiric voice hygiene treatment includes avoiding irritants and frequent coughing or throat-clearing, air humidification, avoidance of large meals, caffeine, alcohol and spicy foods. This case, which occurs in a tobacco abuser, has no apparent cause. As such, diagnostic testing is more important than conservative therapy with home air humidification (A) in ruling-out a more serious cause. Referral to a speech-language therapist (C) is recommended for recurrent laryngitis in professional singers or speakers. Tympanometry (D) is a test commonly used in the evaluation of middle ear function in adolescents. It offers no direct help in diagnosing the source of hoarseness.

A 68-year-old African-American man presents to your office with complaints of urinary frequency, hesitancy and nocturia. Digital rectal exam reveals asymmetric areas of induration and nodules. Which lab finding is most consistent with the diagnosis? Elevated blood urea nitrogen Elevated serum prostate specific antigen Presence of protein on urinalysis Presence of white blood cells on urinalysis

Correct Answer ( B ) Explanation: In the United States, prostate cancer is the most frequently diagnosed type of cancer in men after skin cancer. Prostate cancer is seen more commonly in African-American men and the likelihood of developing this type of cancer increases with age. It is uncommon in men younger than fifty. Risk factors include a family history of prostate cancer, cigarette smoking and a diet high in animal fat. Most patients diagnosed with prostate cancer are asymptomatic and the cancer is found on digital rectal exam (DRE) or because of an elevated serum prostate specific antigen (PSA). Men with risk factors or suspicious physical exam findings on DRE should have a workup initiated that includes evaluation of serum PSA. Diagnosis is made with biopsy. Treatment decisions are determined after discussion with the patient about the severity of disease and quality of life related to treatment side effects. Options include active surveillance, prostatectomy, radiation therapy and hormonal therapy. Elevated blood urea nitrogen (A) is indicative of kidney dysfunction and may be seen in patients with kidney disease, urinary tract infection (UTI), and those taking certain medications. The presence of protein (C) or white blood cells (D) on urinalysis are findings seen in patients with UTIs. UTIs are much less common in men than women due to urethral length, antibacterial substances in the prostatic fluid and a dry periurethral environment that hinders the growth of bacteria.

Screening for lung cancer with low-dose computed tomography of the chest is indicated for which one of the following patients with a 30-pack-year smoking history? A 50-year-old current smoker A 60-year-old who quit smoking 10 years ago A 75-year-old who quit smoking 20 years ago An 85-year-old current smoker

Correct Answer ( B ) Explanation: It is recommended to undergo screening for lung cancer with low-dose computed tomography in adults aged 55 to 80 who have a 30 pack-year smoking history and currently smoke or who have quit within the past 15 years. Screening should be discontinued once a person has not smoked for 15 years or develops a health problem that substantially limits life expectancy or the ability or willingness to have lung surgery. The risk of lung cancer increases with age and cumulative exposure to tobacco smoke and decreases with time since quitting smoking. The National Lung Screening Trial enrolled adults aged 55 and older. An adult aged 50 years (A) did not meet the criteria for the study and therefore should not undergo screening. A smoker that quit 20 years ago (C) does not qualify for the screening. In fact, screening should be discontinued for an individual who has not smoked in over 15 years. Screening is only recommended in smokers up to 80 years of age (D).

Which of the following is most consistent with alcohol withdrawal? A Agitation and increased appetite B Delirium, hallucinations, tremor C Depressed mood with diarrhea D Dilated pupils, runny nose and watery eyes

Correct Answer ( B ) Explanation: Major alcohol withdrawal, also known as delirium tremens, occurs in less than 5% of alcoholics in withdrawal. Delirium tremens is usually preceded by minor withdrawal symptoms, but may occur at any time in the course of withdrawal. The delirium often begins 3 to 4 days after the last drink and is characterized by a marked change in sensorium with agitation, visual hallucinations, and severe disorientation. Most alcoholics who withdraw from alcohol experience minimal symptoms, such as sleep disturbance or anxiety. A small number may have tremulousness, agitation, diaphoresis, and cognitive impairment. The tremors or shakes typically begin 12 to 14 hours after a period of heavy drinking and are usually noted in the early morning. Other symptoms of withdrawal include nausea, vomiting, poor oral intake, sweats, and anxiety. Seizures during alcohol withdrawal tend to occur as one isolated seizure or a brief cluster of seizures. Treatment is supportive (eg. intravenous hydration, nutritional supplementation, close monitoring) as well as benzodiazepines for control of psychomotor agitation and seizures. Agitation and increased appetite (A) along with depressed mood and fatigue is seen in a patient withdrawing from cocaine. Depressed mood with diarrhea (C), dilated pupils, runny nose and watery eyes (D) are all seen with opiate withdrawal.

Olanzapine is a commonly used medication in patients with schizophrenia. Which of the following statements is true regarding the rapid dispersing formulation (wafer) of olanzapine? At overdose levels it will cause bradycardia It is considered to be bioequivalent to olanzapine tablet Rarely associated with orthostatic hypotension The risk of extrapyramidal side effects is higher than with haloperidol

Correct Answer ( B ) Explanation: Olanzapine is an atypical antipsychotic used in the treatment of schizophrenia, bipolar disorder, and acute agitation. It affects multiple receptors (serotonin, dopamine, muscarinic, adrenergic, histamine), but has the greatest affinity as a blocker of the serotonin (5HT2A) receptor and dopamine (D2) receptor. It is available in oral, rapid dispersible (wafer), and intramuscular preparations. In the ED, the rapid dispersible preparation is particularly valuable to manage acutely agitated patients due to its rapid absorption through the oral mucosa and resulting in rapid somnolence. Both oral and rapid dispersible (wafer) preparations are considered bioequivalent. Overdose leads to tachycardia, rather than bradycardia (A) due mainly to muscarinic blockade. Olanzapine is associated with a lower incidence of extrapyramidal effects (D) than haloperidol. Olanzapine is associated with orthostatic hypotension (C) due to the adrenergic (alpha) receptor blockade.

Which of the following is an absolute contraindication for the mumps, measles, rubella (MMR) vaccine? Current minor upper respiratory infection Pregnancy Sibling with a history of febrile seizures Sibling with autism

Correct Answer ( B ) Explanation: Pregnancy is an absolute contraindication for the mumps, measles, rubella (MMR) vaccine. Other absolute contraindications include women intending to become pregnant within one month, immunocompromised patients, patients on high dose steroids and patients with severe allergies to previous MMR vaccination or neomycin. Patients with current or recent minor acute illnesses (A) should receive vaccination. No research has been found to link autism (D) to MMR vaccination, although parents may need to be educated about this concern as misinformation is prominent in the media. Patients with a personal or family history of febrile seizures (C) should proceed with caution and be monitored, but this is not an absolute contraindication.

Which of the following features can differentiate myocardial infarction from pericarditis? Chest pain ECG with reciprocal changes ECG with ST-segment elevations T wave flattening

Correct Answer ( B ) Explanation: Reciprocal ST-segment depressions should never be seen in patients with pericarditis and an ECG with this finding should always be assumed to be from myocardial ischemia or infarction. There is no single best test for pericarditis but clinicians rely on the ECG as the best tool. Early in the disease, the ECG findings of acute pericarditis can mimic those of acute myocardial infarction (MI). The differentiation is critical as timely treatment of MI improves outcomes. Additionally, thrombolytic therapy should not be given to patients with pericarditis as it may precipitate hemorrhage into the pericardial space. Chest pain (A), ST-segment elevation (C) and T wave flattening (D) can be seen in both pericarditis and MI.

A 29-year-old man with a history of HIV presents with shortness of breath and fever. He has a productive cough but denies hemoptysis. You obtain the chest radiograph seen above. Which of the following is true regarding the patient's diagnosis? Elevated LDH is common Spread is by the hematogenous route Steroids should be administered prior to antibiotics if the PaO2 is <80 Trimethoprim-sulfamethoxazole is the treatment

Correct Answer ( B ) Explanation: The chest radiograph demonstrates miliary tuberculosis (TB), or acute disseminated tuberculosis. The term miliary was first used to describe the pathologic lesions seen on radiography that appeared as small millet seeds. Miliary TB occurs when the host is unable to contain a recently acquired or a dormant TB infection. The condition was mostly seen in young children after primary infection but now is more common in the elderly and in persons infected with HIV. Spread of the mycobacteria occurs through the hematogenous route, which leads to the multisystem nature of miliary TB. Clinically, patients develop many of the similar signs and symptoms of active pulmonary TB—fever, weight loss, anorexia, and weakness. Hemoptysis is uncommon. The classic miliary pattern seen in the radiograph is present in approximately 50% of cases. Hyponatremia is sometimes seen from the development of SIADH. Mortality rates are higher than for other forms of TB, which is likely due to a delay in treatment. Elevated LDH (A) commonly occurs in Pneumocystis jiroveci pneumonia. Also with Pneumocystis jiroveci pneumonia, steroids (C) should be administered prior to antibiotics when the PaO2 is <70 or the A-a gradient is >35. Trimethoprim-sulfamethoxazole (D) is the treatment for Pneumocystis jiroveci pneumonia. Treatment of miliary TB is the same as pulmonary TB.

A female infant is born via spontaneous vaginal delivery to a 23-year old G3P3 with no prenatal care at 37 weeks gestation. On physical exam she is found to be small for gestational age and has hepatosplenomegaly, jaundice, and a grade II/VI continuous cardiac murmur. An ophthalmologic exam reveals micro-opthalmia and cataracts. Her CBC reveals thrombocytopenia. Which maternal prenatal screening labs would have indicated these exam findings? Group B strep cultures Rubella immunity status Varicella Zoster screening VDRL screening

Correct Answer ( B ) Explanation: The major complication of rubella is its teratogenic effects when pregnant women contract the disease, especially in the early weeks of gestation. The virus can be transmitted to the fetus through the placenta and is capable of causing serious congenital defects, abortions, and stillbirths. Fortunately, because of the successful immunization program initiated in the United States in 1969, rubella infection and congenital rubella syndrome rarely are seen today. Rubella immunity status is a part of the routine prenatal screening that is performed at the first prenatal visit. Sensorineural hearing loss is the most common manifestation of congenital rubella syndrome. Ocular abnormalities including cataract, infantile glaucoma, and pigmentary retinopathy occur in approximately half of children with congenital rubella syndrome. Both eyes are affected in almost all patients, and the most frequent findings are cataract and rubella retinopathy. Congenital heart disease is present in half of infants infected in the first 2 months' gestation. Other findings in congenital rubella syndrome include the following: intrauterine growth retardation, CNS abnormalities, hepatosplenomegaly, jaundice, blueberry muffin skin spots, bone lesions, hematologic disorders such as anemia and thrombocytopenia. Routine prenatal screening for syphilis (D) remains the most important factor in identifying infants at risk of developing congenital syphilis. Early manifestations of congenital infection vary and involve multiple organ systems. About 60% of infants born with congenital syphilis are asymptomatic at birth. Symptoms develop within the first 2 months of life. In symptomatic infants, the most common physical finding, reported in almost 100% of cases, is hepatomegaly. The other common findings are skeletal abnormalities, rash, and generalized lymphadenopathy. Neonatal varicella (C) is rare because varicella titers are always drawn as part of the prenatal routine labs. These babies appear healthy at birth but develop signs and symptoms of varicella 5-10 days after delivery. In addition, infants born to VZV-seronegative mothers are at risk for severe varicella if infected during the first few days of life. Conversely, infants who have varicella lesions at birth or within the first 5 days of life (and who, thus, received transplacental maternal VZV antibody prior to delivery) are unlikely to have severe disease. Perinatally acquired varicella is characterized by visceral organ involvement (including lung, liver, and CNS disease), with a mortality rate of 30% GBS cultures should be obtained during 35-37 weeks gestation. Group B Streptococcus (A) GBS is one of the leading causes of neonatal sepsis. Neonates present with fever, lethargy, poor feedings and GBS can causes meningitis and pneumonia.

A 19-year-old man presents with multiple painful ulcers on his penis. Genital examination reveals multiple purulent-based, sharply defined, circular ulcers on the shaft of the penis. He also has tender, enlarged, inguinal lymph nodes. Which of the following organisms is the most likely cause of his symptoms? Borrelia burgdorferi Haemophilus ducreyi Plasmodium falciparum Treponema pallidum

Correct Answer ( B ) Explanation: The patient has chancroid. This is an infection caused by the gram-negative bacteria Haemophilus ducreyi. It is a sexually transmitted infection. One week after exposure, the patient develops painful pustules which progress to painful ulcers with purulent bases. Chancroid is usually accompanied by painful inguinal lymphadenopathy. Diagnosis is usually clinical after syphilis and herpes have been ruled out. Appropriate antibiotics include azithromycin, ceftriaxone, ciprofloxacin, and erythromycin. Borrelia burgdorferi (A) is the spirochete that causes Lyme disease. Treponema pallidum (D) is the spirochete that causes syphilis. If this patient had syphilis, one would expect to find a solitary, painless ulcer with a clean base. Plasmodium falciparum (C) is the protozoa that causes malaria.

A 45-year-old man presents to the ED with pain and swelling of his right elbow. He does not recall any specific trauma. On exam, the patient has full passive range of motion with no pain. The elbow is slightly warm to touch. Which of the following is the most appropriate management for this patient? Aspirate fluid for cell count and culture Ice and NSAIDs Incision and drainage Posterior mold splint

Correct Answer ( B ) Explanation: The patient has olecranon bursitis. This is a painful inflammation of the bursa and is often secondary to local trauma, infection, or related systemic illness. Many times, patients cannot recall a specific traumatic event. Occupational risks related to excessive frictional forces include those who frequently place their elbows on a hard desk or a carpenter who spends long periods on his knees. Commonly involved sites include the olecranon, the greater trochanter of the femur, shoulder, and knee. Unless there is concomitant infection, treatment is conservative with ice and NSAIDs. There should also be temporary cessation of the activity that led to bursitis, or protective equipment should be worn, such as kneepads, for people doing prolonged work on their knees. Aspiration of the bursa (A) is an option if an infection is suspected. Septic bursitis typically presents with warmth and erythema over the joint and is sometimes difficult to differentiate from septic arthritis. Incision and drainage (C) are not recommended in bursitis unless there is associated development of an abscess. A posterior mold splint (D) is not indicated in bursitis. Some patients may experience comfort from a sling, but rarely is immobilization ever indicated.

A 17-year-old high school football player is seen in your clinic with neck pain after practice. He reports the pain started after he tackled a teammate and was associated with tingling in both arms. What is next best step in management? Analgesia and observation Cervical spine immobilization Neurosurgical referral X-ray of cervical spine

Correct Answer ( B ) Explanation: This patient is reporting neck pain and neurological symptoms after football practice, which is concerning for a cervical spinal cord injury. Initially, the patient's spine should be immobilized while simultaneously assessing the airway, breathing, and circulation (ABCs). If the injury occurs on the football field, the helmet and shoulder pads should not be removed to ensure spine stabilization. Initial immobilization should be done by stabilizing the head and neck with two hands. If the player is not supine, he should be log-rolled into supine position for the initial assessment. If there is airway compromise, the face mask should be removed. In a clinically stable patient immobilization is done by placing the patient in supine position and placing the neck in a rigid cervical collar. The most common mechanism of injury in adolescents is via axial loading, which can cause a cervical fracture of C1 (Jefferson fracture). This is most likely to occur in football, when a player runs headfirst into another player, and diving, when a diver's head hits the bottom of the pool. Analgesia and observation (A) are not indicated in someone with concern for spinal cord injury. Pain medications can be administered after the spine is immobilized and the patient is stabilized. Neurosurgical referral (C) may be necessary, depending on the severity of the injury; however, primary steps should be taken to prevent further injury by spine immobilization. An X-ray (D) and other imaging modalities of the spine will likely be done after the initial assessment and stabilization to evaluate severity of the injury.

A patient is being evaluated for dyspnea and lower extremity swelling. On physical exam the patient has jugular venous distention with inspiration and 2 + pedal edema. Hepatojugular reflex is present. The patient has clear lung fields and no murmur is appreciated. Which of the following findings would support the diagnosis of constrictive pericarditis over restrictive cardiomyopathy? Cardiomegaly Left bundle branch block Pericardial knock S3 heart sound

Correct Answer ( C ) Explanation: A Pericardial knock is associated with constrictive pericarditis, which is the most likely diagnosis in this patient. Approximately 50% of patients with constrictive pericarditis present with a pericardial knock, which is an accentuated heart sound occurring slightly earlier than a third heart sound. This would not be expected in restrictive cardiomyopathy. Conversely, an audible S3 is frequently present in persons with restrictive cardiomyopathy because of the abrupt cessation of the rapid ventricular filling. Constrictive pericarditis is a rare but disabling condition characterized by impaired filling resulting from restraint of ventricular diastolic expansion by a stiff pericardium. Any cause of pericarditis can lead to the condition. Some common causes include viruses, cardiac surgery, mediastinal irradiation and connective tissue disease. Patients present with dyspnea, fatigue and peripheral edema. Examination may show evidence of right sided heart failure including ascites, pedal edema, hepatojugular reflex and jugular vein engorgement with inspiration, also known as a Kussmaul sign. Pulmonary congestion is absent. Both constrictive pericarditis and restrictive cardiomyopathy can present with signs and symptoms of right sided heart failure making it difficult to distinguish one from the other. Further diagnostic testing and specific physical exam findings can aid in making the correct diagnosis. A S3 heart sound (D) is frequently present in persons with restrictive cardiomyopathy while an accentuated S2 or pericardial knock is associated with constrictive pericarditis. Cardiomegaly (A) on chest x-ray is more suggestive of restrictive cardiomyopathy whereas radiographic evidence of a calcified pericardium strongly suggests constrictive pericarditis. Depolarization abnormalities, such as a left bundle branch block (B) on electrocardiogram strongly favor restrictive cardiomyopathy.

A 29-year-old woman who is two weeks postpartum following an uncomplicated pregnancy and delivery of a full-term infant is brought in by emergency medical services with an ongoing generalized tonic-clonic seizure. Which of the following medications should be administered first? Labetalol Lorazepam Magnesium sulfate Phenobarbital

Correct Answer ( C ) Explanation: A pregnant or recently postpartum patient with new-onset seizure should be considered to have eclampsia. Eclampsia refers to seizures that develop as a complication of severe preeclampsia. The clinical manifestations of preeclampsia are hypertension after 20 weeks of pregnancy plus proteinuria. Severe preeclampsia is evidenced by marked hypertension (blood pressure ≥ 160 mm Hg systolic or ≥110 mm Hg diastolic) with evidence of end-organ dysfunction, such as visual disturbances, mental status changes, pulmonary edema, epigastric or right upper quadrant pain, elevated liver function tests, thrombocytopenia, proteinuria, oliguria, or impaired fetal growth. Most cases of eclampsia occur in the 3rd trimester, with approximately 80% occurring during delivery or within the first 48 hours after delivery, though seizures may occur as late as several weeks postpartum. Seizures are most commonly tonic-clonic and last 60 to 90 seconds. Magnesium sulfate is the drug of choice for eclamptic seizures. A loading dose of 4-6 g of magnesium sulfate should be administered over 15-20 minutes followed by a maintenance infusion of 1-2 g per hour. Most eclamptic seizures terminate with magnesium. Labetalol (A) may be used to control severe hypertension in a patient with preeclampsia or eclampsia, but does not treat seizures. Lorazepam (B) and phenobarbital (D) are second- and third-line choices, respectively, if eclamptic seizures are refractory to magnesium.

A 58-year-old man ran out of his congestive heart failure medications. He presents with significant dyspnea and altered mental status. Examination reveals bibasilar crackles and jugular venous distension. An electrocardiogram shows sinus rhythm with low voltage complexes. Which of the following is most appropriate at this time? Amiodarone Disopyramide Furosemide Ventricular assist device

Correct Answer ( C ) Explanation: Acute or decompensated congestive heart failure may be the result of myocardial infarction, dysrhythmias, dietary noncompliance, medical noncompliance, kidney failure, pulmonary embolism, anemia or toxins. Acute failure typically requires hospitalization and critical care. In-house care is comprised of oxygen, nitrates, and furosemide. Furosemide is a loop diuretic that inhibits the transporter at the loop of Henle in the kidneys leading to free water clearance. Inotropic medications, such as dobutamine and dopamine, may also be necessary if there is evidence of impaired perfusion. Severe cases may require an intraaortic balloon pump or a ventricular assist device. Amiodarone (A) is an antiarrhythmic used in treating acute and chronic dysrhythmias. It is not routinely used in the acute decompensation of heart failure, especially when a patient is in normal sinus rhythm. Disopyramide (B) is used in treating hypertrophic cardiomyopathy and arrhythmias but not heart failure. A ventricular assist device (D) is reserved for cardiac failure that fails medication management, or as a bridge to cardiac transplant.

A 30-year-old daycare worker has a 3-day history of arthralgias, malaise, and a rash. The rash is nonpruritic, maculopapular, irregular, and is located on her thighs and inner aspects of her upper arms. Symmetric synovitis is present in her fingers. Small effusions, warmth, and tenderness are noted in her left wrist and right elbow. No other joints are affected. Which of the following is most likely responsible for this presentation? Adenovirus Measles (rubeola) virus Parvovirus B19 Varicella-zoster virus

Correct Answer ( C ) Explanation: Also known as erythema infectiosum or Fifth disease, parvovirus B19 infection usually is asymptomatic or causes mild, nonspecific, cold-like symptoms. However, several clinical conditions have been linked to the virus. Parvovirus B19 usually infects children and causes the classic "slapped-cheek" rash of erythema infectiosum (fifth disease). The virus also may cause acute or persistent arthralgias and papular, purpuric eruptions on the hands and feet in adults. This infection should be particularly suspected in health-care workers who have frequent contact with children. Parvovirus B19 infection can also trigger an acute cessation of red blood cell production, causing transient aplastic crisis, chronic red cell aplasia, hydrops fetalis, or congenital anemia. Adenovirus (A) is a mild virus that causes fever, conjunctivitis, pharyngitis, rash and can manifest as gastroenteritis. It is prevalent in both the pediatric and adult population and treatment is symptomatic. Measles (rubeola) virus (B) is characterized by cough, coyza, conjunctivits, maculopapular rash and the pathognonic oral enathem known as Koplik spots. Measles virus and adenovirus do not cause arthritis. Varicella-zoster virus (D) also known as chickenpox typically presents with fever, malaise and a widespread vesicular and pruritic rash located on the torso and face. This virus was once common in children before the advent of the varicella vaccine in the United States. Adults, pregnant women, immunocompromised patients and neonates are at high risk of complications from infection with varicella such as pneumonia, hepatitis and neurologic sequelae. Varicella-zoster virus may cause large joint arthritis, but the rash is distinctively vesicular and pruritic.

Which of the following is true regarding botulism? Foodborne botulism develops 10-14 days following toxin ingestion Foodborne botulism is caused by ingestion of a heat-stable toxin (type A) Infantile botulism is the most common form Intestinal colonization is common in foodborne botulism

Correct Answer ( C ) Explanation: First described in the mid-1970s, infantile botulism is now the most common form of botulism in the United States. Children typically present with poor feeding, decreased suckling, loss of facial expression, constipation, and noticeable neck and peripheral weakness—a constellation of symptoms known as "floppy baby syndrome." It occurs in children younger than 12 months of age, with a peak incidence at three months. It has been associated with ingestion of honey, corn syrup, and vacuum or environmental dust. It is caused by the ingestion and subsequent intestinal colonization of Clostridium botulinum spores in the intestinal tract. Foodborne botulism (A and B) occurs primarily in adults within one to two days following the ingestion of preformed heat-labile toxin present in canned foods that have not been properly preserved. Several toxin types (A, B, E, and F) have been identified as causing disease in humans. Adults may initially complain of difficulty swallowing or speaking, diplopia, blurred vision, or ptosis. The hallmark of botulism poisoning is the development of bilateral, symmetrical cranial neuropathies and a descending flaccid paralysis (upper extremities > lower extremities). The muscles of respiration are often affected, and patients can progress to respiratory failure. Intestinal colonization (D) occurs in infantile botulism. This rarely occurs in adults because the presence of normal intestinal flora prevents colonization of the GI tract.

A 19-year-old man presents with pain in his index finger. Which of the following is suggestive of flexor tenosynovitis? Delayed capillary refill Holding the finger fully extended Pain on passive extension Swelling localized to the volar aspect of the finger

Correct Answer ( C ) Explanation: Flexor tenosynovitis is an infection of the flexor tendon sheath of the finger. Most commonly, penetrating trauma to the actual sheath causes this infection although direct spread from other areas of the hand may occur. Staph aureus and streptococci are the most common causative organisms. Kanavel's signs are the four cardinal signs of acute flexor tenosynovitis include: 1) tenderness along the flexor tendon, 2) symmetric swelling of the finger ("sausage finger"), 3) pain on passive extension and 4) flexed posture of the finger. These patients need antibiotic therapy and admission to the hospital for possible operative drainage. Delayed capillary refill (A) may be seen with significant swelling in a finger infection but is not a hallmark sign for flexor tenosynovitis. Typically the pain of the infection is exacerbated when the finger is fully extended and as a result patients avoid holding the finger fully extended (B). More often, the finger will be flexed and on passive extension by the clinician the pain is exacerbated. Swelling is not localized to the volar aspect of the finger (D) but is more symmetric and uniform along the entire finger creating the sausage-like appearance.

A man with dyspnea-on-exertion presents for cardiac evaluation. Physical exam is significant for a left sternal border systolic murmur which is louder during a Valsalva maneuver. An S4 is also appreciated. The ECG shows large QRS complexes. An echocardiogram reveals a decrease in left ventricular chamber volume and normal left atrial measurements. Which of the following is the most likely diagnosis? Aortic stenosis Dilated cardiomyopathy Hypertrophic cardiomyopathy Mitral stenosis

Correct Answer ( C ) Explanation: Hypertrophic cardiomyopathy (HCM) results from left or right ventricular hypertrophy or both. This condition can be primarily caused by autosomal dominant genetic mutations of the cardiac sarcomere genes and myocardial fiber hypertrophy. Secondary causes include aortic stenosis, mitral valve abnormalities, coronary heart disease and chronic systemic hypertension. Most patients are asymptomatic upon presentation, however, the common clinical manifestations are dyspnea, angina and dysrhythmia. Decreased chamber volume and increased ventricular wall thickness are the key echocardiographic findings in HCM. Furthermore, the ECG typically shows left ventricular hypertrophy, T-wave inversion and large QRS complexes. A harsh, left sternal border, systolic, crescendo-decrescendo murmur which is worse with Valsalva maneuver is quite typical of HCM. Aortic stenosis (A) is associated with a right, not left, sternal border pansystolic murmur that is decreased, not increased, during a Valsalva maneuver. S3, normal or thin ventricular wall and enlarged ventricular chamber volume are more common with dilated cardiomyopathy (B). S4, thick ventricular wall and decreased ventricular chamber volume are more common with hypertrophic cardiomyopathy. Mitral stenosis (D) produces a diastolic, not systolic, murmur, and is associated with increased left atrial size and pressure.

A 14-year-old girl with a 2-year history of bulimia nervosa presents to the emergency department with hematemesis. You suspect an upper gastrointestinal hemorrhage, and order an emergent endoscopy. Which of the following will most likely be found with this diagnostic test? H.pylori infection Hematochezia Mallory-Weiss tear Meckle's diverticulum

Correct Answer ( C ) Explanation: Mallory-Weiss tears are the cause of 1-15% of all upper GI bleeding. These tears are described as longitudinal mucosal lacerations of the distal esophagus, especially occurring at the gastroesophageal junction. Although 25% of cases are idiopathic, these tears commonly occur as a result of repeated retching, vomiting (as in bulimia nervosa), hiccuping, coughing, straining, abdominal trauma, gastric prolapse into the esophagus and cardiopulmonary resuscitation. When suspected, immediate upper endoscopy is used to confirm diagnosis. 80-90% of tears will resolve spontaneously, and most heal with therapy in 2 days. 40-70% will however require blood transfusions, and 10% may progress into hemorrhagic shock. Since this pathology is a cause of GI bleeding, initial management includes cardiopulmonary stabilization with admission and monitoring, serial hemoglobin testing, transfusion therapy if necessary, fluids, proton-pump inhibition and antiemetics. More definitive treatment may involve endoscopic heat, epinephrine or sclerosing agents, angiotherapy or surgical oversewing of the tear.` Hematochezia (B) is the term for bloody stools, and is more commonly a sign of lower GI bleeding than upper GI bleeding. However, if the upper GI bleed is rapid, hematochezia may be present, but it a sign from the rectum, not a diagnostic finding on endoscopy. H.pylori infection (A) is common with gastric and duodenal ulcer disease, which can cause upper GI bleeding. However, it is not diagnosed with endoscopy, but rather with serology, stool antigen and urease testing. Meckle's diverticulum (D) is a congenital diverticulum of the small intestine present at birth. It is a vestigial remnant of the omphalomesenteric duct. Most are asymptomatic, while some present with hematochezia and melena.

Which of the following maneuvers tests the meniscus of the knee? Anterior drawer test Lachman test McMurray test Posterior drawer test

Correct Answer ( C ) Explanation: Meniscal injuries occur frequently in patients with sudden rotary or extension-flexion motions. The menisci have no sensory nerve fibers, and the pain that results after these injuries is from irritation of the ligaments near the joint line. Several symptoms suggest the presence of a meniscal tear including: joint line pain, joint effusion, locking, and giving way of the knee. The McMurray test is performed with the patient supine and the hip and knee flexed. To check the medial meniscus, the examiner palpates the posteromedial joint line with one hand while the other hand grasps the foot. The leg is externally rotated to trap the medial meniscus, and the knee is slowly extended. Conversely, the lateral meniscus is examined with the clinician palpating the posterolateral joint line while internally rotating the leg. A painful click, popping, or thud felt in early extension is considered abnormal. It is 53% sensitive for a meniscal injury. The Anterior Drawer (A) and Lachman tests (B) are used to assess anterior cruciate ligament (ACL) instability. The anterior drawer test is 62% sensitive for ACL disruption. With the affected knee flexed at 45 degrees, the clinician pulls the tibia forward away from the femur. The test is positive if the tibia is displaced more than 6 millimeters compared to the opposite leg. The Lachman test is more sensitive for ACL disruption (84%). The patient's affected knee is flexed 30 degrees. The clinician stabilizes the femur with one hand while the other hand grasps the tibia. The tibia is displaced in a forward direction. The test is positive if the tibia can be displaced greater than 5 millimeters anteriorly compared to the opposite leg. The Posterior Drawer test (D) assesses the posterior cruciate ligament. It is similar to the anterior drawer test, but instead of a forward displacement, the tibia is displaced posteriorly. It is 55% sensitive for PCL disruption.

A 33-year-old woman with no medical problems presents with chest pain and shortness of breath. The symptoms worsened over the previous two days and increase with exertion. The patient denies cough, but last week reports fevers, chills, coughing and myalgias. Her ECG demonstrates sinus tachycardia without ST segment changes. Vital signs are T 100.7°F, HR 120, BP 108/60, RR 16, 100% saturation on room air. Which of the following is the most likely diagnosis? Acute coronary syndrome Mediastinitis Myocarditis Pulmonary embolism

Correct Answer ( C ) Explanation: Myocarditis is typically caused by a viral infection, most commonly coxsackie B virus, adenovirus, parvovirus B19 and Echovirus. Trypanasoma cruzi (Chagas disease) is the most common etiology worldwide. Myocardial necrosis occurs likely as a result of direct invasion of the offending organism as well cytotoxic effects of the host's immune system. Individuals develop flulike symptoms and in adults chest pain and shortness of breath. One of the hallmark signs of myocarditis is tachycardia out of proportion to fever. Depending on the time of presentation, patients may have symptoms of heart failure as the left ventricular ejection fraction is impaired as a result of the myocarditis. Troponin is often elevated as the disease progresses. The gold standard for diagnosis is endocardial biopsy. Management is supportive. ACE inhibitors help reduce myocardial inflammation. Question: In a patient traveling from Central America, what is a leading cause of myocarditis? Acute coronary syndrome (A) is a possibility in this patient but is less likely given her age and absence of medical history. It is also atypical for a person with acute coronary syndrome to have a fever and viral symptoms. Mediastinitis (B) may present with fever, chest pain and shortness of breath. However, there is often a history of either severe retching or a procedure that caused esophageal perforation. Patients with air in the mediastinum from a perforation may have Hamman's sign, a crunching sound on auscultation. Pulmonary embolism (D) does cause chest pain and shortness of breath. Classically the chest pain is pleuritic and flulike symptoms are not associated with the diagnosis. Patients with a pulmonary embolism may have a low-grade fever.

A 6-year-old girl returns from summer swimming-camp with otalgia and difficulty hearing. Otoscopic examination reveals otorrhea and auditory canal erythema. The tympanic membrane is normal. Palpation of the auricle and surrounding bone is nontender, however, lateral pulling of the auricle reproduces significant pain. Which of the following is the most likely diagnosis? Auricular chondritis Mastoiditis Otitis externa Otitis media

Correct Answer ( C ) Explanation: Otitis externa is defined as inflammation of the external auditory canal. It is frequently due to bacterial pathogens, such as staphylococci, Proteus and Pseudomonas, however, some cases are the result of infection with fungi like Candida and Aspergillus. Risk factors include cerumen impaction, instrumentation, high humidity environments and water immersion (i.e., common in swimmers). Symptoms include otalgia, otorrhea, pruritis and decreased hearing. Pain with traction on the tragus or auricle (pinna) is a key sign, and is usually accompanied by erythema, edema and otorrhea. Treatment includes avoiding water, debridement, topical corticosteroids and antibiotics. Left untreated, otitis externa can become invasive, leading to osteomyelitis, cranial nerve palsies and death, or can spread to the auricle, causing chondritis. Auricular chondritis (A) may present as a complication of untreated otitis externa. If present, palpation of the auricle would be exquisitely tender. Mastoiditis (B) is a possible complication of otitis media. A hallmark exam finding is severe mastoid tenderness during palpation of the external ear's surrounding temporal bone. Otitis media (D) does not typically present with tender auricle traction. Furthermore, tympanic examination is usually abnormal.

An 82-year-old woman with osteoporosis falls against her kitchen table. She presents with acute right shoulder pain, proximal right upper extremity edema and near-absent active shoulder joint range of motion. Which of the following is the most appropriate next step in management? Obtain a scapular-Y radiograph Passive range of motion measurement Perform a distal neurovascular examination Perform a Hawkins testing

Correct Answer ( C ) Explanation: Proximal humerus fractures are most commonly seen in elderly patients. The most common type of fracture occurs at the surgical neck (the epiphyseal-diaphyseal junction, just inferior to both tuberosities). As with any fracture, it is imperative to assess distal neurovascular supply with motor, sensory, and pulse examination. Complications from shoulder injuries include injury to the brachial plexus and vascular such as the axillary artery. The Hawkins test (D), which usually follows Neer's testing of the shoulder, is used to evaluate subacromial impingement in the setting of rotator cuff syndrome or tear, the results of which are most likely noncontributory given this presentation. Passive range of motion measurements (B) will most likely be noncontributory and difficult to obtain. Radiographs (A) occur only after the clinician is assured that no acute neurovascular compromise is present.

A 36-year-old Guatemalan male migrant farm worker complains of a left eye mass that is seen in the image above. Which of the following is the most likely diagnosis? Cataract Pinguecula Pterygium Xanthelasma

Correct Answer ( C ) Explanation: Pterygia are fibrovascular proliferations thought to be triggered by ultraviolet light exposure. They occur more commonly in young males from hot, dry climates. Pterygia are characterized by their shape which is widest at the bulbar conjunctiva and narrowest at the cornea. Visual impairment may occur in large pterygia, but most are asymptomatic. Surgical removal is necessary once vision is impaired. Pinguecula (B) is a raised yellowish white mass on the bulbar conjunctiva similar to pterygia, but they do not extend to the cornea. Xanthelasma (D) are also yellow masses around the eye, but they occur from elevated plasma lipid level and are seen on the canthus of the eyelid. Cataracts (A) appear as an opacity in the lens, typically in the elderly.

An asymptomatic 4-year-old boy presents for a routine check-up. On examination you hear a systolic heart murmur that is heard best in the lower precordium and has a low and short tone. It does not radiate and decreases in intensity with inspiration. The remainder of the examination is normal. Which one of the following is the most likely diagnosis? Eisenmenger's syndrome Mitral stenosis Still's murmur Venous hum

Correct Answer ( C ) Explanation: Still's murmur is a benign murmur of childhood. It can occur at any age, but is most commonly heard between ages 2 and 8 years. The murmur usually resolves spontaneously by adulthood. It has no association with physiologic or anatomic abnormalities. It is low-grade mid-systolic murmur localized near the apex to lower left sternal border. It decreases in intensity with inspiration, sitting up, or standing. Most pathologic murmurs do not change significantly with standing except the murmur associated with hypertrophic cardiomyopathy. The sound is often described as musical or having a relatively pure tone in quality or squeaky. Because they are low pitched, they are best heard with the bell of the stethoscope. The cause of Still's murmur is unknown, but it may be due to vibrations in the chordae tendinae, semilunar valves, or ventricular wall. Mitral stenosis (A) causes a diastolic murmur and Eisenmenger's syndrome (B) involves multiple abnormalities of the heart that cause significant signs and symptoms, including shortness of breath, cyanosis, and organomegaly. Venous hum (D) consists of a continuous low-pitched murmur caused by the collapse of the jugular veins and their subsequent fluttering, and it worsens with inspiration or diastole.

A 51-year-old man walks into a movie theater and experiences acute onset of right eye pain associated with nausea, vomiting, and cloudy vision. Which of the following is expected during the ophthalmologic exam? Cherry-red spot in the macular area Deep anterior chamber Intraocular pressure >21 mm Hg Miotic pupil

Correct Answer ( C ) Explanation: The patient has acute angle-closure glaucoma. This condition results in optic nerve damage from increased intraocular pressure. In patients with a narrow anterior chamber angle, reduced illumination (like entering a dark movie theater) causes mydriasis. Subsequently, folds of the peripheral iris can block the angle, which prevents aqueous humor outflow. This leads to a rapid elevation of intraocular pressure causing ocular pain, a hazy cornea, ciliary flush, a firm globe, and optic nerve damage if the pressure is not promptly relieved. An intraocular pressure >21 mm Hg is considered elevated. Pressures can elevate quickly to >60 mm Hg. The higher the pressure, the quicker damage occurs to the optic nerve and the poorer the prognosis. The diagnosis is often delayed due to the associated symptoms of nausea, vomiting, and abdominal pain. Treatment involves reducing aqueous humor production with IV acetazolamide, topical beta-blockers (timolol), and topical alpha-agonists (apraclonidine). Topical miotic agents (pilocarpine) are used to reverse the angle closure. Topical steroids help to reduce inflammation. Hyperosmotic agents (mannitol, glycerol) can also be administered for further reduction in intraocular pressure. The funduscopic finding of a pale retina with a cherry-red spot in the macular area (A) is consistent with central retinal artery occlusion. This condition is associated with sudden unilateral vision loss that is painless. It is caused by a thrombotic plaque or embolus of the central retinal artery. A deep anterior chamber (B) is protective against acute angle-closure glaucoma. Individuals with a narrow chamber are at increased risk. The pupil in acute angle-closure glaucoma is most commonly fixed and mid-dilated, rather than miotic (D). Miotic pupils are associated with opiate use, cholinergic toxicity, and pontine strokes.

A 42-year-old man presents with facial pain. He reports pain over his cheeks and forehead with associated fever for the last 24 hours. On inspection of his nasal passages you note inflamed turbinates with green discharge. He is tender over palpation of the frontal and maxillary sinuses. Which of the following is the most appropriate course of action? CT scan of the sinuses ENT consultation Oxymetazoline and pseudoephedrine Prescription for amoxicillin/clavulanic acid

Correct Answer ( C ) Explanation: This patient has rhinosinusitis. Viral upper respiratory infections and allergic rhinitis are the most common causes of acute rhinosinusitis. Additional risk factors are ciliary immobility or dysfunction, structural abnormalities, and immunocompromise. Patients with viral sinusitis are at risk of developing bacterial sinusitis as a consequence of the viral infection. Clinically patients with acute rhonisinusitis develop mucopurulent nasal discharge, facial or sinus pain, and nasal congestion. Symptoms of acute sinusitis typically progress over the first several days and spontaneously resolve after 7 to 10 days. It is difficult to distinguish clinically between a viral and bacterial infection in the first several days of illness and antibiotic therapy is not recommended at this time. Management focuses on symptomatic treatment with pain management and decongestant therapy. Antihistamines may provide some benefit for patients with allergic rhinosinusitis. Decongestant therapy is available topically with agents like oxymetazoline. Systemic therapy includes pseudoephedrine. Saline nasal irrigation is beneficial for all forms of acute rhinosinusitis. Topical and systemic steroids are no longer recommended for acute sinusitis. A CT scan of the sinuses (A) is not necessary in this patient. Imaging is indicated when there are concerns for complications of cellulitis (e.g. cavernous sinus thrombosis, abscesses, orbital involvement) or invasive fungal infections. ENT consultation (B) is not necessary for uncomplicated sinusitis. A prescription for amoxicillin/clavulanic acid (D) is not indicated in the first several days of illness because of the likelihood this is viral. Without improvement after symptomatic therapy or progression to chronic sinusitis, antibiotics are indicated.

A 23-year-old man presents with swelling and pain of his elbow. On examination, he has swelling over the olecranon. He has full range of motion at the joint. Vital signs are normal. What management is indicated? Arthrocentesis Aspiration Compression dressing and nonsteroidal antiinflammatory drugs Incision and drainage

Correct Answer ( C ) Explanation: This patient presents with a noninfectious olecranon bursitis that should initially be managed with a compression dressing, ice and NSAIDs. The olecranon bursa frequently becomes inflamed as a result of repetitive minor trauma. Patients will typically present with pain, swelling and tenderness over the olecranon. The inflammation is often obvious on physical examination. Although active range of motion may be limited by pain, patients should have full passive range of motion as the joint is not involved. Noninfectious bursitis should have minimal or no warmth and erythema. Noninfectious bursitis should be treated conservatively with compression dressings, ice, NSAIDs and orthopedic follow up as well as avoidance of the inciting trauma. Arthrocentesis (A) should only be performed if it is believed that the joint is involved. Aspiration (B) of the bursa may be diagnostic and therapeutic in septic bursitis but does not play a role in noninfectious bursitis. Incision and drainage (D) is reserved for recurrent or refractory bursitis.

A 66-year-old woman presents with a rash to the right side of her forehead that began 2 days ago. She describes it as painful, blistering, and weeping. On exam, you note the rash seen above. Which of the following is the most likely diagnosis Acne rosacea Contact dermatitis Herpes zoster Impetigo

Correct Answer ( C ) Explanation: Zoster ophthalmicus is due to reactivation of latent varicella virus in the dorsal root ganglia. It is characterized by a painful rash described as papulovesicular eruptions on an erythematous base along a dermatome. The lesions are often preceded by pain in a dermatomal distribution 2-3 days before the characteristic rash appears. Lesions often involve the face, mouth, eyes, ears, or tongue when branches of the trigeminal nerve are affected. Involvement of the tip of the nose is referred to as Hutchinson's sign and often signals concurrent ocular involvement of the nasociliary branch. Ramsey-Hunt Syndrome refers to involvement of the external auditory canal and development of an ipsilateral facial palsy. Treatment of zoster ophthalmicus involves analgesics, anti-inflammatories, and anti-viral medications (acyclovir or valacyclovir). The most common complication secondary to herpes zoster is post-herpetic neuralgia (chronic pain) of in the area of the involved dermatome. Acne rosacea (A) is an inflammatory disorder characterized by erythema, telangiectasias, and pustules primarily affecting the central face. It is primarily seen in women over the age of 30, but can be seen in men also. Contact dermatitis (B) is a rash caused by an exposure to either an irritant or allergen. The rash can be either chronic or acute and can have many different appearances depending on the type and length of exposure. Impetigo (D) is a superficial infection of the skin caused by S. aureus or beta-hemolytic streptococci. It is characterized by honey-colored, crusted lesions

In a diabetic patient with new microalbuminuria, which of the following is the recommended first line treatment for the prevention of kidney disease? Amlodipine Blood pressure control to less than 120/70 Hemoglobin A1c less than 8.0 Low protein diet

Correct Answer ( D ) Explanation: A low protein diet has been shown to decrease urine albumin excretion and improve glomerular filtration rate (GFR). This may lower the rate of progression to end-stage renal disease and death compared with a normal-protein diet. Blood pressure control is also an important consideration for diabetic patients with microalbuminuria. ACE inhibitors are recommended for first line treatment of blood pressure. Amlodipine (A) may be used to help control blood pressure, but is not considered first line. Major guidelines suggest that the goal blood pressure in patients with diabetes mellitus is less than 140/90 mmHg. Evidence does not support blood pressure control less than 120/70 (B) as it has not been shown to make a difference on the rate of end-stage renal disease, the need for dialysis, nonfatal myocardial infarction, nonfatal stroke or cardiovascular death. Glycemic control is also recommended, but with a goal A1c of less than 7.0, making hemoglobin A1c less than 8.0 (C) incorrect.

A middle-aged man presents with a complaint of "my head is expanding," stating that it seems like his hat does not fit him anymore. Which of the following signs suggest a condition of excess growth hormone? Exophthalmos and thin, brittle skin Facial rounding and erythema Hyperpigmentation and orthostatic hypotension Prognathism and macroglossia

Correct Answer ( D ) Explanation: Acromegaly is a condition of excess growth hormone post puberty. The most common cause is a growth hormone producing pituitary gland adenoma. Symptoms include soft tissue enlargement, with macroglossia, increased ring, hat or shoe size, polyarthritis and hyperhidrosis. The pituitary mass effect can produce headaches and visual field defects. Common physical findings are frontal bossing (enlarged frontal skull bones), enlarged jaw and protruding jaw (prognathism), enlarged tongue (macroglossia), nose thickening, hirsutism, enlarged digits, oily skin and skin tags. Treatment includes somatostatin, bromocriptine and growth hormone antagonists. Exophthalmos and thin skin (A) are common signs of hyperthyroidism. A rounded, erythematous face (B) (moon fascies) is commonly seen in patients with Cushing's syndrome. Primary adrenal insufficiency, Addison's disease, is marked by hyperpigmentation and orthostatic hypotension (C).

Which finding on synovial fluid analysis is most consistent with the diagnosis of pseudogout? Elevated synovial leukocyte count Multiple gram-positive cocci Negatively birefringent urate crystals Positively birefringent calcium pyrophosphate dihydrate crystals «

Correct Answer ( D ) Explanation: Calcium pyrophosphate crystal deposition disease, also referred to as pseudogout, is a common crystal-induced arthropathy that generally affects the large joints. Pseudogout has a similar clinical presentation to gout, but the etiology is different. Pseudogout may be idiopathic, especially in the elderly. It may also be caused by trauma, hyperparathyroidism, hemochromatosis, and medications that cause hypomagnesemia such as loop diuretics or proton pump inhibitors used in peptic ulcer disease. Patients present with acute onset of severe pain, inflammation and edema in the knees, ankles, elbows or wrists. Pseudogout is generally monoarticular, but may present in multiple joints as well. Diagnosis is by synovial fluid analysis, so arthrocentesis is necessary for patients with monoarticular arthritis. Synovial fluid analysis will show positively birefringent calcium pyrophosphate dihydrate crystals, which confirms the diagnosis. Initial treatment for pseudogout is with nonsteroidal anti-inflammatory drugs (NSAIDs) or colchicine. Elevated synovial leukocyte count (A) is seen in septic arthritis. Multiple gram-positive cocci (B) aspirated from an inflamed joint points to the diagnosis of gonococcal arthritis, caused by infection with the sexually transmitted Neisseria gonorrhoeae bacteria. Negatively birefringent urate crystals (C) seen on synovial fluid analysis are consistent with the diagnosis of gout.

A 73-year-old man presents with painless vision loss in the right eye. Which of the following on funduscopic examination is most characteristic of central retinal vein occlusion? Cherry-red fovea Pale retina Papilledema Retinal hemorrhages

Correct Answer ( D ) Explanation: Central retinal vein occlusion occurs as a result of thrombus formation in the retinal vein. This can occur in the setting of multiple causes including mechanical compression, sluggish circulation, vasculitis, and hypercoagulability. As the central retinal vein is occluded, there is increased resistance to venous flow in the retinal venous system. The impaired bloodflow can increase pressure causing retinal hemorrhages and the classic "blood and thunder" appearance on funduscopic examination. In addition to these findings, the retinal veins may appear dilated and tortuous with macular and optic disc edema (unilateral). Ultimately the pressure increase leads to retinal ischemia and subsequent vision loss. The degree of vision loss depends on how much venous obstruction actually occurs. The cherry red fovea (A) is seen in cases of central retinal artery occlusion. Central retinal artery occlusion essentially causes an ischemic infarct of the retina. In this case, examination reveals a pale retina (B) with the fovea standing out as a cherry-red spot. Papilledema (C) is usually a bilateral physical examination finding where the optic disc is edematous with blurred edges. This is representative of conditions that cause increased intracranial pressure.

A 35-year-old woman with a history of schizophrenia presents to your office with a complaint of auditory hallucinations and suicidal ideation. The patient has tried several second-generation antipsychotics without relief from her symptoms. You decide to prescribe clozapine. Which of the following tests should be conducted on a recurrent basis to monitor this patient? Electrocardiogram Fasting blood sugar Fasting lipid panel White blood cell and absolute neutrophil count

Correct Answer ( D ) Explanation: Clozapine may be considered in patients who are not responding to first-line treatment for schizophrenia or who are continually engaging in self-harm or suicidal behavior. Patients receiving clozapine are entered into a national registry and white blood cell monitoring for agranulocytosis must occur at specific intervals. While agranulocytosis is rare (approximately 0.8% of patients), it is life-threatening and therefore requires stringent monitoring. Other side effects of clozapine can include seizures, myocarditis, weight gain and sedation. Because of clozapine's unique side effect profile, assessment of general health and cardiovascular status is required prior to initiating treatment. This includes a baseline electrocardiogram (A), fasting blood sugar (B) and fasting lipid panel (C). However, these tests are not required as part of the ongoing lab work for patients taking clozapine. Cardiovascular status is monitored with weekly blood draws for eosinophil count, troponins and sedimentation rate or C-reactive protein.

A previously healthy 42-year-old man presents to your office with questions about screening for colon cancer. He has no family history of colon cancer, but heard that he should start getting a colonoscopy every 5 years starting at age 40. Which of the following recommendations do you provide? He does not need screening because he has no family history of colon cancer He should be scheduled for a colonoscopy as soon as possible He should start screening at age 50 and continue indefinitely He should start screening at age 50 and continue until age 75

Correct Answer ( D ) Explanation: Colorectal cancer is the third most common type of cancer for both men and women in the United States. The United States Preventative Services Task Force recommends colon cancer screening for average-risk individuals starting at age 50 years through age 75 years. Screening may occur using one of three modalities: high-sensitivity fecal occult blood testing annually, sigmoidoscopy every 5 years with fecal occult blood testing every 3 years, or colonoscopy every 10 years. Patients with a first-degree relative (parent, sibling or child) who has a history of colorectal cancer or adenomas should be screened earlier than age 50 years. Screening for these patients is determined on an individualized basis and generally starts at age 40 years or 10 years younger than the age the affected relative was diagnosed. All adults regardless of screening risk should be screened for colorectal cancer starting at age 50 (A). Patients without a first-degree relative who has been diagnosed with colorectal cancer should follow the USPSTF guidelines and start colorectal cancer screening at age 50 years. There is no need for colonoscopy prior to this timeframe (B). Previous guidelines did not give a recommendation for when to end screening (C), but the newest guidelines now indicate that screening should stop at age 75 years.

A three-year-old girl presents with headache, fatigue, anorexia, and myalgias after returning from a trip to the Dominican Republic. You notice multiple mosquito bites on physical examination. You suspect dengue fever. What is the treatment of choice for this diagnosis? Acyclovir Ceftriaxone Ciprofloxacin Supportive care

Correct Answer ( D ) Explanation: Dengue fever is an RNA virus transmitted to humans by infected mosquitos in the tropical regions of Southeast Asia, Mexico, Central America, Africa, and the Caribbean. Children younger than 15 years old have increased severity and mortality. Associated morbidity and mortality is secondary to hemorrhage from thrombocytopenia, platelet dysfunction, or disseminated intravascular coagulation. Symptoms include abrupt onset of high fever, retro-orbital pain, headache, myalgias, arthralgias, mucosal bleeding, and petechiae. Associated fatigue, nausea, vomiting, and abdominal pain are common. Laboratory testing commonly reveals thrombocytopenia, leukopenia, elevated liver transaminases, and elevated hematocrit. Supportive care is the treatment of choice. Confirmatory testing is rarely obtained but can be obtained by sending serum for reverse transcriptase-PCR of dengue virus or ELISA dengue IgM. Acyclovir (A) is an antiviral used in the treatment of herpes encephalitis. It is not effective in the treatment of dengue. Ceftriaxone (B) is a cephalosporin commonly used to treat meningitis. Since the question reveals the suspected cause of the patients symptoms as dengue, an antibacterial would not be effective. Ciprofloxacin (C) is often used in travelers with suspected bacterial diarrhea and gastroenteritis. It has no role in the treatment of dengue fever.

A patient complains of a 6-month history of tinnitus and vertigo that occurs with nausea, vomiting and a sensation of "ears full of water." These symptoms occur intermittently in attacks at a frequency of once or twice a week. He denies head trauma, headache, photophobia or focal neurologic deficits. Otoscopic examination reveals no external auditory canal or tympanic membrane abnormalities. Weber and Rhinne testing supports unilateral sensorineural hearing loss. In addition to diet changes, trigger avoidance and diuretics, which of the following is strongly recommended for the initial management of these symptoms? Endolymphatic sac decompression Labyrinthectomy Pyridostigmine Vestibular rehabilitation

Correct Answer ( D ) Explanation: Meniere's disease is caused by an idiopathic excess of inner ear fluid (endolymphatic hydrops) that causes dilation of and breaks in the membranous labyrinth, thus allowing perilymph and endolymph to mix. Postulated associations include head trauma, otitis media and upper respiratory tract infection, sequellae of herpes virus infection and use of aspirin, alcohol, tobacco and excess salt and caffeine. The classic clinical picture is intermittent vertigo, intermittent sensorineural hearing loss, aural fullness and tinnitus. Nausea and vomiting frequently accompany the attacks. Management begins with stress reduction, trigger avoidance, alcohol and tobacco avoidance, and a low-salt diet, which can be augmented with prescription diuretics. Physiotherapy, namely vestibular rehabilitation, is a treatment mainstay. In this modality, a physical therapist uses a specific program of balance, postural and gaze stabilizing exercises to habituate the brain to vertiginous symptoms via neuronal plasticity re-training. Allergy desensitization may be necessary, and trans-tympanic membrane corticosteroid injections have been used. Surgery is entertained if the above fail, or the patient deteriorates. Options include endolymphatic sac shunt-decompression (A), vestibular neurectomy and ultimately chemical or excisional labyrinthectomy (B). Pyridostigmine (C), a reversible cholinesterase inhibitor used in treating myasthenia gravis, acts as a cholinergic agonist, which worsens symptoms of dizziness. Comparatively, a cholinergic antagonist, like scopolamine, is recommended in the treatment of Meniere's.

A 22-year-old woman with regular menstrual cycles presents with symptoms concordant with premenstrual syndrome. In evaluating the large differential of these symptoms, which of the following serum laboratory tests is recommended? Estrogen Follicle stimulating hormone Human chorionic gonadotropin Thyroid-stimulating hormone

Correct Answer ( D ) Explanation: Premenstrual syndrome includes more than 150 behavioral, physical, cognitive and emotional symptoms. The most common behavioral symptom is mood swings, although anxiety, depression, increased appetite, anhedonia and sensitivity are common. A key historical point in making the diagnosis of premenstrual syndrome is the absence of symptoms during the follicular phase of the menstrual cycle. It is important to remember that other common conditions, such as thyroid disorders and hypercortisolism, can mimic the behavioral and affective changes of premenstrual syndrome. Thyroid abnormalities, both hyperthyroidism and hypothyroidism, can result in such mood symptoms. Although the initial laboratory evaluation of suspected premenstrual syndrome is limited, a serum thyroid-stimulating hormone level is recommended. Serum sex steroids (A) and gonadotropins (B) are similar in women with or without premenstrual syndrome. A woman with current regular menstrual cycles is unlikely to be pregnant, as such, a human chorionic gonadotropin (C) level is unrevealing.

An elderly woman underwent cataract surgery last week. Today, she complains of central vision loss and peripheral flashing lights. This patient is likely to emergently require which of the following procedures? Blepharoplasty Canaloplasty Laser assisted in-situ keratomileusis Pneumatic retinopexy

Correct Answer ( D ) Explanation: Retinal detachment refers to a peeling away of the retina from its underlying supportive connective tissue. Risk factors include prior cataract surgery, head or eye trauma, family history and severe myopia. Considered an emergency, the most common symptoms are acute central vision loss, an acute increase in the numbers of floaters, peripheral vision flashes of light (photopsia), and eye heaviness. Fundoscopic exam with photography is the recommended evaluation when detachment is suspected. 85% of cases improve with one surgical procedure, while the remaining 15% improve with two or more surgical procedures. Common surgeries include cryo therapy and laser photocoagulation, scleral buckle therapy, pneumatic retinopexy and vitrectomy. Blepharoplasty (A) is an eyelid plastic surgery used to remove excessive skin or fat ("eyelift"). Canaloplasty (B) is used to improve trabecular meshwork drainage in a patient with increased intraocular pressure. Laser assisted in-situ keratomileusis (LASIK) (C) is used to correct refractive vision issues, not retinal detachment.

A 7-year-old boy presents with five days of abdominal pain, arthralgias, and a rash of the lower extremities. Mild diffuse abdominal tenderness is noted on examination. The left knee and right ankle are swollen and tender without warmth or erythema. The rash consists of purple, non-blanching macules and papules of both legs and thighs. Which of the following is indicated? Abdominal ultrasound to evaluate for intussusception Antinuclear antibody to evaluate for rheumatologic disease Complete blood count to evaluate for thrombocytopenia Urinalysis to evaluate for glomerulonephritis

Correct Answer ( D ) Explanation: The above patient's constellation of a purpuric rash, abdominal pain, arthralgia, and arthritis is concerning for Henoch-Schönlein Purpura (HSP), also known as IgA vasculitis. HSP is a small vessel vasculitis that typically follows upper respiratory infection by one to three weeks. The first symptom of the vasculitis is typically the rash, which may begin as macules or urticaria prior to evolving into palpable purpura. The abdominal pain of HSP results from vasculitis of the gastrointestinal vasculature and may rarely precede the development of the HSP rash. The gastrointestinal vasculitis may also serve as a lead point for intussusception, which presents with severe, intermittent, colicky abdominal pain. The arthritis of HSP is typically migratory and oligoarticular and most commonly affects the joints of the lower extremities. While swelling and tenderness of the affected joints is common, erythema and warmth are typically absent. The arthritis of HSP may also precede the rash by a few days. Glomerulonephritis is also a common feature of HSP. Thus, this patient requires a urinalysis to evaluate for glomerulonephritis, specifically to evaluate for blood, casts, and protein. Renal manifestations of HSP vary widely. While 20-30% of children develop gross hematuria, others develop a range of manifestations from asymptomatic hematuria or non-nephrotic range proteinuria to acute renal insufficiency. The child must be screened for development of renal disease over at least six months following presentation, as renal manifestations of HSP can continue to develop after other signs and symptoms have resolved. 91% of children who develop renal disease will present within six weeks and 97% by six months. If urinalysis is normal six months after presentation, there is no further risk of permanent renal damage. An abdominal ultrasound to evaluate for intussusception (A) is not necessary in most patients with abdominal pain. This child's diffuse, mild abdominal pain is consistent with gastrointestinal vasculitis.However, if the pain becomes severe and colicky, or if the child develops current jelly stools, an abdominal ultrasound should be immediately obtained. An antinuclear antibody (ANA) to evaluate for rheumatologic disease (B) is not necessary in a patient with HSP. ANA is a non-specific marker of autoimmune disease such as Systemic Lupus Erythematosus, Juvenile Idiopathic Arthritis, and Dermatomyositis, as well as Hashimoto's thyroiditis, Graves' disease, autoimmune hepatitis. ANA is not sufficiently specific to diagnose a particular rheumatologic disease, and it may also be positive in individuals without rheumatologic disease. For example, ANA is more likely to be positive in individuals who have a family member with autoimmune disease, even if the individual remains unaffected. ANA may also become positive years prior to the onset of autoimmune disease. Moreover, a positive ANA may be induced by medications, infection, or malignancy. A complete blood count to evaluate for thrombocytopenia (C) is not necessary in a patient with HSP. While purpura may be caused by thrombocytopenic conditions such as Immune Thrombocytopenic Purpura, this patients constellation of acute onset of purpura, abdominal pain, and arthritis is more consistent with HSP.

A 25-year-old man presents for evaluation of fever and cough. He reports last week that he was diagnosed with influenza. In the last 2 days he developed a worsening cough productive of large amounts of sputum. Vital signs are T 101°F, HR 98, BP 120/60, RR 18, and 95% oxygen saturation on room air. His chest X-ray demonstrates a lobar infiltrate in the left lower lobe. Which of the following would you most likely expect to see on the patient's Gram stain? Gram negative bacilli Gram negative diplococci Gram positive bacilli Gram positive cocci in clusters

Correct Answer ( D ) Explanation: The patient had a recent influenza infection and now presents with a lobar infiltrate. Staphylococcus aureus pneumonia is classically associated with causing post-influenza bacterial pneumonia. On Gram stain this is seen as Gram positive cocci in clusters. Gram negative bacilli (rods) (A) include a number of organisms common in pneumonia including Klebsiella and Pseudomonas. Gram negative diplococci (B) include Neisseria sp., Morexella sp., Acinetobacter sp., Haemophilus sp., and Brucella sp. Gram positive bacilli (rods) (C) should raise concern for possible pulmonary anthrax. Gram positive cocci in chains (Streptococcus pneumoniae) is also commonly associated with post-influenza pneumonia.

A 26-year-old woman presents two days after an operation for recurrent sinusitis. Her husband states that she has been confused since she got "the flu" yesterday. Her vitals are temperature 39.5°C, HR 115, BP 95/70, and oxygen saturation is 99% on room air. On exam, she is disoriented and has a diffusely hyperemic, blanching rash. She has a surgical dressing covering her nose. What is the next step in management? Administer broad-spectrum antibiotics Obtain a CT scan of the sinuses Perform a wound culture Remove the surgical dressing

Correct Answer ( D ) Explanation: The surgical dressing must be removed to ensure there is no nasal packing or other foreign body present that could serve as a precipitant of toxic shock syndrome (fever, hypotension, diffuse erythroderma, multisystem organ dysfunction). Toxic shock syndrome is commonly associated with postsurgical dressings as well as vaginal foreign bodies (classically, extended-use tampons). Toxic shock syndrome toxin-1 (TSST-1) producing strains of S. aureus cause the infection, with the toxin serving as a superantigen that leads to overstimulation of T-lymphocytes and subsequent massive, unregulated cytokine release. Patients often report a prodrome of flu-like symptoms including headache, myalgias, vomiting, and diarrhea. Antibiotics (A) prevent recurrence but do not affect the outcome of the acute illness. However, starting an antistaphylococcal antibiotic early is standard practice. There is evidence that the addition of clindamycin or linezolid offers an additional benefit of decreasing production of TSST-1. A wound culture (C) has limited utility in this scenario. A blood culture may be useful but should be obtained after the nidus of infection is removed. All diagnostic testing, including imaging (B), should be performed after searching for and removing the likely source of toxin.

A 52-year-old man states he took his blood pressure and it was elevated to 180/100 mm Hg. He states that he missed his regular dose of antihypertensive medication because he was traveling for business and returned home today. His blood pressure now is 176/102 mm Hg. The patient is otherwise asymptomatic and has a normal physical exam. What is the most appropriate action? Admission for blood pressure monitoring Obtain an ECG Reduce mean arterial pressure by 25% with an intravenous antihypertensive Resume outpatient medication

Correct Answer ( D ) Explanation: This patient has asymptomatic hypertension. Without any symptoms or signs of end organ damage, there is no acute intervention necessary. Patients with elevated blood pressure may have vague symptoms (headache, weakness, fatigue). There is no evidence to support the acute reduction of blood pressure in these patients. The physician may elect to administer a dose of the patient's usual medication or discharge with a plan for resumption of these medications at home. Since the patient in the above clinical scenario has a known diagnosis of hypertension and did not recently take his medications, it is appropriate to have him resume the same medication at the same dose. Patients who have an elevated blood pressure despite taking their antihypertensives will need to be assessed by their primary care physician for alteration of their regimen. Patients do not require admission for blood pressure monitoring (A) when they are asymptomatic. Patients with hypertensive emergency will need admission to the hospital and may require an intensive care unit setting. An ECG (B) is required only if the patient has chest pain or has some other reason suggestive of a dysrhythmia or coronary ischemia. In cases of hypertensive emergency and end-organ damage, the goal is to reduce mean arterial pressure by 25% (C) in the first hour. Normalization of blood pressure should occur over the next 8 to 24 hours. Intravenous medications are preferable for this situation since they have a rapid onset and are titratable.

A 5-year-old boy with sickle cell disease presents with a limp, which his mother noticed 2 days ago. On exam, the patient is well appearing. Vital signs are significant for fever with temperature of 38.1°C. Heart rate and blood pressure are normal. The patient demonstrates an antalgic gait, and there is warmth and tenderness to palpation of the soft tissues in the distal thigh. Lab tests show a normal WBC count and elevated CRP and ESR. Plain radiographs of the femur show soft tissue swelling in the distal thigh as well as a periosteal reaction in the distal femur. Knee radiographs are normal. What is the most likely infecting organism? Haemophilus spp. Klebsiella spp Pseudomonas spp. Salmonella spp.

Correct Answer ( D ) Explanation: This patient has osteomyelitis of the distal femur. Osteomyelitis, which is a pyogenic infection of the bone, occurs via hematogenous seeding (most common in children), seeding from a contiguous source of infection (eg, diabetic foot ulcer), or direct inoculation (eg, puncture wound, orthopedic procedure). Children will most often present with acute osteomyelitis (< 2 weeks) with signs of systemic infection such as fever and irritability in addition to localized bone pain and swelling. Adults commonly present with subacute or chronic osteomyelitis, and will have pain, swelling, as well as non-healing ulcers more frequently than they will have fever. Plain radiographs are the initial imaging choice in patients with osteomyelitis, though fewer than one third of patients will have any abnormalities in the first seven to ten days of admission. Periosteal reaction is the earliest osseous finding. The most common infecting organism in osteomyelitis for healthy hosts in all age groups except neonates is Staphylococcus aureus. Patients with sickle cell have an increased risk of osteomyelitis overall, especially from Salmonella spp. Additionally, Salmonella spp. are the most common pathogen identified in sickle cell patients with osteomyelitis. Pseudomonas (C) infection should be considered in the setting where osteomyelitis develops following a penetrating injury through a surface colonized with pseudomonas (e.g. the sole of a shoe). Haemophilus spp. (A) used to be the second most common cause of osteomyelitis, however this is no longer true secondary to the advent of the Haemophilus vaccine. Klebsiella spp. (B) and other anaerobes are rarer causes of osteomyelitis and are more frequently associated with chronic osteomyelitis, as in diabetic patients with non-healing foot ulcers.

A 55-year-old man presents with sudden onset of decreased vision and pain in his right eye, He reports vomiting twice prior to presentation. Examination reveals normal extraocular motions and a mid position pupil that does not react to light. What management should be pursued? Acetazolamide drops Metoprolol intravenous Tetracaine drops Timolol drops

Correct Answer ( D ) Explanation: This patient presents with signs and symptoms concerning for acute angle closure glaucoma and should be immediately treated with a topical beta blocker like timolol. Glaucoma represents an ocular neuropathy caused by increased intraocular pressure. Glaucoma occurs either as the result of increased aqueous humor production (open-angle) or due to decreased drainage of the aqueous humor (closed-angle). Open-angle glaucoma is slower in onset and progressively causes changes in vision. Primary angle closure glaucoma, on the other hand, occurs suddenly and can lead to rapid vision loss. It is typically precipitated by papillary dilation (i.e. going into a dimly lit or dark room). Symptoms of angle closure glaucoma include eye pain, headache, nausea, vomiting and change in vision. The cornea may appear cloudy and the pupil midposition and dilated. The diagnosis can be definitively made by finding an increased intraocular pressure on tonometry. Treatment should be initiated promptly with topical timolol and pilocarpine (a miotic agent). In severe cases, intravenous acetazolamide should be added. Acetazolamide (A) is effective if given intravenously in severe cases. Intravenous beta blockers (B) do not play a role in management. Tetracaine (C) is a topical anesthetic and does not play a role in glaucoma management.

A 22-year-old man who just returned from a trip to Scandinavia presents to the ED with complaints of severe cramping abdominal pain and diarrhea. The diarrhea was initially profuse and watery and is now bloody. His vital signs are HR 105 beats per minute, RR 18 per minute, BP 110/64 mm Hg, temperature 38.8 °C, and oxygen saturation 99% on room air. He has right lower quadrant tenderness on exam. Fecal occult blood test is positive; wet mount of the stool shows fecal leukocytes. Which of the following is the most likely diagnosis? Salmonellosis Shigellosis Vibrio parahaemolyticus infection Yersinia enterocolitis

Correct Answer ( D ) Explanation: Yersinia enterocolitica gastroenteritis is a relatively infrequent cause of enteritis in the United States, but it is more common in Scandinavia and Europe. Infection results from ingestion of contaminated food or drink. Patients develop fever and diarrhea two to six days post-exposure accompanied by severe cramping abdominal pain. One historical clue is that the diarrhea is initially profuse and watery and later becomes bloody. In young adults, particularly adolescents, an ileocecitis may develop and present as pseudoappendicitis. While fecal leukocytes increase the suspicion of a bacterial etiology, definitive diagnosis is made by stool culture. Mild to moderate infections are self-limited, but severe infections can be treated with antibiotics (fluoroquinolone, TMP-SMX). Salmonellosis (A) is one of the most common causes of foodborne illness in the United States. Depending on the specific species, infection can result in a variety of clinical syndromes, including gastroenteritis, septicemia, typhoid fever, and an asymptomatic carrier state. Infection usually results in stools that are watery, foul, and brownish-green often with blood. Shigellosis (B) (aka, bacillary dysentery) is highly infectious and is primarily spread by fecal-oral transmission. Patients present with high fever, crampy abdominal pain, and diarrhea that is watery and greenish-yellow. Bloody mucoid stools (dysentery) occur in approximately one-third of patients. A CBC with a leukocytosis and marked left shift (absolute band count > 800) is characteristic of shigellosis. Vibrio parahaemolyticus (C) is the most common cause of foodborne bacterial enteritis in Japan. It is acquired by ingestion of raw or improperly prepared seafood (oysters, clams, shrimp, crabs). The diarrhea is usually explosive and associated with vomiting, cramps, and dysentery.

Question: What is the Sokolow-Lyon electrocardiographic definition of left ventricular hypertrophy?

Answer: S wave in V1 + R wave in V5 or V6 ≥ 35 mm. Rapid Review Hypertrophic Cardiomyopathy Asymmetric LV septal wall hypertrophy → outflow obstruction Autosomal dominant (familial form) Young patient Exertional syncope Sudden cardiac death S4 gallop Midsystolic murmur (↑ as preload ↓) Rx: ßBs or CCBs

Question: What conditions are relative contraindications to the use of clozapine?

Answer: Seizure disorder, pre-existing neutropenia, and heart disease. Rapid Review Clozapine Agranulocytosis Seizures Myocardits Increased mortality in elderly patients with dementia-related psychosis

Question: Which bacterial gastroenteritis is associated with febrile seizures?

Answer: Shigella. Rapid Review Yersinia Enterocolitica Patient with a history of eating undercooked pork Complaining of RLQ pain, fever, vomiting and then diarrhea Labs will show fecal WBCs and RBCs

Question: What is the upper limit of normal diameter for the different segments of bowel?

Answer: Small bowel (3 cm), colon (6 cm), cecum (9 cm)—the 3/6/9 rule. Rapid Review Large Bowel Obstruction CRC > diverticulitis, volvulus, strictures High-pitched bowel sounds Abdominal distension NGT, surgery

Question: What is the most effective way to decrease the mortality and morbidity associated with lung cancer?

Answer: Smoking cessation.

Which of the following conditions, in which laws and social norms are repetitively violated, is the most common precursor to antisocial personality disorder? Conduct disorder Malicious compliance behavior disorder Oppositional defiant disorder Passive-aggressive personality disorder

Correct Answer ( A ) Explanation: Conduct disorder is a disorder of children and adolescents. It is characterized by a persistent and repetitive pattern in which the subject violates the basic rights of others. Usually the subject repetitively violates laws and social norms. As with other behavior or conduct disorders, it is important first to rule out an underlying substance abuse disorder as the cause of changing behaviors and actions. The childhood-onset subtype, before the age of 10 years, results in longer lasting and more pervasive behaviors than the adolescent-onset (after the age of 10 years) subtype. Children in the childhood-onset subtype have a greater incidence of ADHD, academic problems, family dysfunction, violence and aggression. Conduct disorder is the best predictor of antisocial personality disorder, which cannot be diagnosed until the subject is 18 years old. Malicious compliance behavior (B) describes a person who strictly follows orders from management authorities in an intentional act of harming said management, business or employees. This industrial behavior can be thought of as sabotage. Passive-aggressive personality disorder (D) is a pervasive pattern of negativistic attitudes and passive resistance to demands for adequate performance of work or social obligations. Characteristics include procrastination or a deliberate task avoidance, hostile joking and resentment. Oppositional defiant disorder (C) is characterized by a pervasive pattern of disobedience, anger, stubbornness, hostility and defiant behavior toward authority figures. Law violation is not a common trait. It cannot be diagnosed if a child matches criteria for conduct disorder.

A 10-year-old boy presents with a two day history of sore throat, fever and headache. He denies cough, significant rhinnorhea or head congestion. Physical exam is remarkable for enlarged, erythematous tonsils with a pharyngeal whitish exudate. He has marked lymphadenopathy over his anterior cervical lymphoid chain. What is the most likely diagnosis? Mononucleosis Sinusitis Strep throat Tonsillolithiasis

Correct Answer ( C ) Explanation: "Strep throat" caused by Group A Beta-hemolytic streptococcus (Streptococcus pyogenes or GAS) is a common etiology of acute pharyngitis especially in children ages 5 to 15. It is characterized by inflammation of the pharynx or tonsils (tonsillar exudates) associated with symptoms of fever, malaise and sore throat, as well as the absence of other URI symptoms such as nasal congestion and cough. Cervical lymphadenopathy is often found on exam, as is a whitish exudate over the pharynx and tonsils. A rapid streptococcal antigen test is recommended in order to determine if treatment with antibiotics is warranted, as other conditions which do not require antibiotic treatment may mimic streptococcal pharyngitis. A throat culture to rule-out GAS infection is recommended in children if rapid antigen testing is negative (90% sensitivity), in order to limit transmission and prevent complications such as rheumatic fever. Other complications of strep throat may include acute glomerulonephritis, peritonsillar abscess, bacteremia, sinusitis and pneumonia. Penicillin-based antibiotics (benzathine penicillin IM or oral penicillin VK) are the treatment of choices. For penicillin allergic patients, azithromycin is an alternative. Mononucleosis (A) is an infection caused by Epstein-Barr virus and is most common in 15 to 24 year-olds. Patients usually experience a several day prodrome of fever, chills, malaise and anorexia followed by the onset of throat pain, fever and lymphadenopathy. The diagnosis is supported by heterophile (Monospot) antibody testing. Treatment is supportive and the illness is generally self-limited, though some patients may experience residual symptoms of malaise and fatigue for months following initial diagnosis. Sinusitis (B) refers to inflammation of the mucous membranes lining the paranasal sinuses. Patients present complaining of cough, facial tightness, headache, reduced ability to smell and nasal congestion with purulent nasal and postnasal discharge lasting 7-10 days. Sinusitis is usually caused by a virus, leading to decreased clearance of secretions within the mucosa and entrapment of bacteria which may lead to secondary bacterial infection. The three most common bacterial agents involved in sinusitis are Haemophilus influenzae, Streptococcus pneumoniae and Moraxella catarrhalis. Treatment of viral sinusitis is supportive; antibiotics are indicated only if the clinical course strongly suggests bacterial secondary infection. Tonsillolithiasis (D) is a condition caused by chronic inflammation of the tonsils in which small concretions develop within one or both of the tonsils. Symptoms of this condition may include halitosis, foreign body sensation, dysphagia and odynophagia, otalgia, and neck pain, though many patients are asymptomatic. Definitive treatment is surgical removal, though not required if symptoms are not bothersome.

A 50-year-old woman with a history of mitral stenosis secondary to rheumatic fever presents with atrial fibrillation. She does not have a history of heart failure, hypertension, diabetes mellitus or previous stroke or transient ischemic attack. What is the most appropriate management for this patient? Aspirin Dabigatran No anticoagulation Warfarin

Correct Answer ( D ) Explanation: Warfarin is the most appropriate management. This patient has mitral stenosis and requires anticoagulation with warfarin regardless of CHA2DS2 - VASc score. Validated risk factors for thromboembolism in patients with atrial fibrillation include mitral stenosis, previous thromboembolism, heart failure, systolic dysfunction, diabetes, hypertension, presence of a mechanical heart valve and older age. In patients without significant valvular disease, the mostly commonly used method to determine choice of thromboprophylaxis is the CHA2DS2 - VASc score. Those with a score of 0 are at low risk and anticoagulation is a clinical decision. A score of 1 is low to moderate risk and antiplatelet or anticoagulation therapy should be considered. A score of 2 or more is considered moderate to high risk and anticoagulation is recommended. Patients with mitral stenosis who have chronic atrial fibrillation have a stroke risk that may be as high as 7-15 % per year. It is recommended that patients with mitral stenosis and atrial fibrillation be started on warfarin, and it should be continued indefinitely to decrease the risk of systemic thromboembolism. The goal INR is between 2.0 to 3.0. Aspirin (A) is not appropriate because the presence of mitral stenosis necessitates anticoagulation with warfarin. No anticoagulation (C) is not appropriate as this patient has a risk factor that require further therapy. Newer oral anticoagulants, such as dabigatran (B), are now approved for prevention of systemic embolism in adults with nonvalvular atrial fibrillation. However, the use of warfarin in patients with mitral stenosis is recommended because patients with significant mitral valve disease were excluded from the trials of these newer agents.

Question: What is a bubo?

Answer: A large, unilateral fluctuant lymph node which can develop in chancroid.

Question: What late skin finding is commonly associated with staphylococcal toxic shock syndrome?

Answer: Desquamation of palms and soles 1-2 weeks after onset of symptoms. Rapid Review Toxic Shock Syndrome S. aureus Tampons, nasal packs, wounds Hypotension, high fever, diffuse erythroderma, multisystem organ dysfunction Identify/remove source

Question: Which patients are more likely to develop an invasive otitis externa due to Pseudomonas?

Answer: Diabetics. Rapid Review Otitis Externa Swimming, moisture MC bacterial etiology: Pseudomonas Malodorous discharge Pruritis Pain with palpation of tragus/pinna Non-purulent rx: acetic acid with hydrocortisone solution Edema rx : neomycin/polymyxin/hydrocortisone

Question: What is the choroidal tubercle?

Answer: A granuloma in the choroid of the retina that is specific for disseminated TB. Rapid Review Pulmonary Tuberculosis (TB) RFs: immunodeficiency, immigrant, close contact Latent/primary TB: asymptomatic Active/reactivation TB: fever, night sweats, weight loss, productive cough, hemoptysis Erythema nodosum Primary TB CXR: Ghon focus Active/reactivation TB: upper lobes, cavitary lesions Dx: sputum smears for acid-fast bacilli (AFB), sputum/tissue culture for AFB (gold standard) PPD: gold standard for latent TB dx Latent rx: 9 months of INH Primary rx: Rifampin, INH, Pyrazinimde, Ethambutol (RIPE)

Question: What is Ewing sarcoma?

Answer: A marrow malignancy, common in the second decade, that causes lysis and osteomyelitis.

Question: What is Adie's pupil?

Answer: A pupil with parasympathetic denervation that constricts poorly to light but reacts better to accommodation. Gives the appearance of anisocoria. Rapid Review Acute Angle-Closure Glaucoma Patient will be entering a dark room or movie theater Complaining of acute unilateral painful vision loss, vomiting, and seeing halos around lights PE will show cloudy cornea and fixed mid-dilated pupil Labs will show ↑ IOP ( > 21 mm Hg) Treatment is topical ßBs, carbonic anhydrase inhibitors, steroids, miotics

Question: What is the treatment of cauda equina syndrome?

Answer: Emergent decompression (a surgical emergency). Rapid Review Cauda Equina Syndrome Patient will be someone with a history of trauma, malignancy, epidural abscess or hematoma Complaining of acute onset of lower back pain with weakness and numbness PE will show urinary retention, saddle anesthesia, decreased rectal tone Diagnosis is made by MRI or CT myelogram Most commonly caused by a herniated disc Treatment is operative decompression

Question: What are some associated symptoms in gonococcal pharyngitis?

Answer: GU symptoms (dysuria, discharge), joint pain.

Question: Which groups of patients are at increased risk of developing herpetic whitlow?

Answer: Health-care workers (e.g. physicians, nurses, dentists, and dental assistants) and nail salon workers.

Question: Chronic congestive heart failure is managed with which outpatient medications?

Answer: ACE-inhibitors, beta-blockers and diuretics, with consideration of aldactone, nitrates and inotropes. Rapid Review Acute Decompensated Heart Failure Exertional dyspnea, orthopnea, paroxysmal nocturnal dyspnea, pitting edema S3 ↑ BNP CXR: cardiomegaly, cephalization, Kerley B lines, effusions Most useful study: echo Treatment: BiPAP: ↑ oxygenation, ↓ work of breathing, ↓ preload/afterload Nitroglycerin: ↓ preload/afterload Furosemide: diuresis Hypotension without signs of shock: dobutamine (may worsen hypotension) Severe hypotension with signs of shock: norepinephrine (↑ systemic vascular resistance, ↑ HR, ↑ BP, ↑ myocardial O2 demand)

Question: What is found as a predisposing risk factor in patients with Mallory-Weiss tears?

Answer: Hiatal hernia. Rapid Review Mallory-Weiss Syndrome Patient will have a history of drinking alcohol and forceful vomiting Complaining of hematemesis Diagnosis is made by upper endoscopy Caused by an incomplete tear in the esophagus mucosa

Question: What class of medications is used in the short term treatment for alcohol withdrawal?

Answer: Benzodiazepines. Rapid Review Ethanol Withdrawal Autonomic hyperactivity (↑ HR, ↑ BP, diaphoresis) Visual, tactile hallucinations Seizures Delirium tremens: autonomic hyperactivity, psychosis, peaks 2-5 days after cessation Tremulousness → hallucinations → seizures → delirium tremens Rx: BZDs

Question: Which antihypertensive agent is preferable for a hypertensive emergency caused by a pheochromocytoma?

Answer: Intravenous phentolamine is an alpha-blocker which can be used to block the catecholamine effects of a pheochromocytoma. Rapid Review Asymptomatic Hypertension MC causes of BP elevation: pain, anxiety No workup indicated Outpatient follow-up

Question: What pulmonary sequela may occur with dengue fever?

Answer: Increased capillary permeability allows pleural effusions to form as a sequela. Rapid Review Dengue Fever Patient with a history of recent travel to tropical regions Complaining of "breakbone fever" - high fever with a biphasic pattern, myalgias, and backache Labs will show thrombocytopenia, elevated LFTs and leukopenia Most commonly caused by Aedes aegypti mosquito Treatment is supportive

Question: In which population is Klebsiella pneumonia most commonly seen in?

Answer: COPD, Alcoholics and the elderly. Rapid Review Bacterial Pneumonia S. pneumonia: most common, rusty colored sputum, rigors, gram+ paired lancets Klebsiella: alcoholics, currant jelly sputum, bulging fissures, S. aureus: IVDA, postinfluenza, elderly, gram+ cocci in clusters H. influenzae: COPD, gram negative pleomorphic rods Pseudomonas: cystic fibrosis, nursing home resident and cyanosis Health care associated pneumonia: pseudomonas, MRSA Outpatient, healthy: macrolide or doxycycline Outpatient, comorbidity: respiratory tract fluoroquinolone (RTF) Inpatient: RTF ICU: antipneumococcal ß-lactam (ceftriaxone or cefotaxime) + either azithromycin or an RTF

Question: In a patient traveling from Central America, what is a leading cause of myocarditis?

Answer: Chagas' disease caused by Trypanosoma cruzi. Rapid Review Myocarditis Leads to dilated cardiomyopathy, CHF Sudden death in young adults Idiopathic > viral (Coxsackie) Viral prodrome Positive troponin ST-segment elevation ECHO shows global hypokinesis Supportive management, possible transplant

Question: The greater tuberosity is the insertion point of which 3 rotator cuff muscles?

Answer: Supraspinatus, infraspinatus and teres minor. Rapid Review Proximal Humerus Fracture Neer classification: based on number of displaced (> 1 cm) or angulated (> 45°) parts Most common fracture: one part fracture Orthopedic consultation for displaced fractures

Question: Retinal detachment of which area favors a poor prognosis?

Answer: The macula. Rapid Review Retinal Detachment Ocular emergency Painless loss of vision Floaters, flashing lights, curtain lowering sensation Retina appears hazy gray with white folds

Question: What is the treatment for rubella infection?

Answer: Treatment is supportive. Provide vision screening and hearing screening for asymptomatic newborns.

Question: True or false: A prostate specific antigen level of zero guarantees the absence of prostate cancer? a

Answer: True.

Question: How long should breastfeeding continue?

Answer: Until it is no longer mutually desired by the mother and child. Rapid Review Mastitis Patient will be a breastfeeding mother Complaining of breast erythema, tenderness, fever Most commonly caused by Staph. aureus Treatment is warm compresses, dicloxacillin, amoxicillin Comments: continue breastfeeding to avoid progression to abscess

Question: What organisms are typically implicated in septic bursitis?

Answer: Usual skin flora. Rapid Review Olecranon Bursitis Swelling, pain ↓ Active ROM Full passive ROM NSAIDs

Question: What is the most likely lesion in an individual with a yellow mass on the upper eyelid with a history of hyperlipidemia?

Answer: Xanthelasma. Rapid Review Pterygium "Surfer's eye" Benign collagenous growth Medial side Risk factors: sunlight, wind, sand, dust

Question: Can an angiotensin II receptor blocker be used in patients with ACE inhibitor intolerance?

Answer: Yes.

A mother calls the office concerned that she was diagnosed with mastitis while she is still breastfeeding her 1-month-old infant. What is the most appropriate recommendation? Continue breastfeeding from both breasts Continue breastfeeding from the contralateral breast Discontinue breastfeeding, as exclusive breastfeeding is only recommended for one month Pump and discard all breast milk until the mastitis is resolved

Correct Answer ( A ) Explanation: Maternal mastitis is common during breastfeeding and is not a contraindication to continued breastfeeding. While the mother is on antibiotics, she should be instructed to continue to fully empty the breast with each feed. The mother should not be instructed to feed only from the contralateral breast (B), as the risk of transmission of infection from the affected breast is negligible. Pumping and discarding breast milk (D) is unnecessary. Finally, discontinuation of breastfeeding (C) is not recommended. The American Academy of Pediatrics recommends exclusive breastfeeding for the first six months of the infant's life.

A woman presents with 30 minutes of double vision, vertigo, difficulty swallowing, and difficulty speaking. During her observation, these symptoms resolve within 1 hour and her neurologic exam returns to normal. Which of the following is the most appropriate diagnosis and the most likely affected artery? Embolic ischemia - anterior cerebral artery Thrombotic stroke - left anterior descending artery Transient ischemic attack - middle cerebral artery Transient ischemic attack - vertebrobasilar arteries

Correct Answer ( D ) Explanation: Transient ischemic attacks (TIAs) are characterized by an abrupt onset of focal neurologic symptoms lasting less than 24 hours and often lasting only 5 to 20 minutes. TIAs suggest impending thrombotic-ischemic stroke. Carotid pathology leads to TIAs demonstrated by hemiparesis, hemisensory, aphasia, confusion and transient monocular blindness. Vertebrobasilar insufficiency leads to TIAs marked by hemiplegia or quadriplegia, varying sensory changes, blindness, hemianopsia, diplopia, vertigo, dysarthria, dysphagia and facial, motor, and sensory change. Investigation includes Doppler ultrasonography of the carotids and vertebrobasilar system. Head CT scan will not show any acute changes. MRA angiography may also be indicated. Selected cases may require endarterectomy, angioplasty or lifelong anticoagulation. Embolic ischemia typically produces symptoms that last longer than 3 hours and anterior cerebral artery (A) lesions do not produce vertigo and diplopia. Thrombotic stroke (B) indicates infarct, which indicates permanent, not temporary or resolved, neurologic defect. Middle cerebral artery (C) lesions do not produce vertigo.


Kaugnay na mga set ng pag-aaral

Nursing Care For IV Therapy Ch. 7

View Set

Chapter 9, 10 ,11 Strategic managment

View Set

Fundamentals I: Exam 1 Study Guide

View Set

Medical Terminology, Chapter 10: Musculoskeletal (Anatomy & Physiology)

View Set